Vous êtes sur la page 1sur 70

MANAGEMENT ACCOUNTING

SuggestedAnswers
FinalExaminationSummer2013




Page1of6
A.1 Effect of change in debt collection policy Rs. in million
Existing sale 15,575
Existing credit sale (80% of 15,575) 12,460
Existing Debtors 2,590
Debtors turnover (2,590 360 12,460) 75 days
Conversion costs (13,770 50 150) 4,590
Fixed cost (20% of 4,590) 918
Contribution margin (%) [(1805 + 918) 15,575] 17.48%

New credit sale as per proposal (92% of 12,460) 11,463
Decrease in credit sales (12,460 11,463) 997

Loss of contribution margin on decrease in sale (17.48% 997) (174.28)
Reduction in bad debts [(4% 12,460) (3% 11,463)] 154.51
Early payment discount to debtors (11,463 40% 1%) (45.85)
Surcharge from late payment [2866 (W-1) 24 45 1,000] 30.95
Savings from decrease in debtors [(2,590 1,337) 15%] 187.95
153.28

Revising the credit policy is feasible

W-1:

Debtors analysis Credit sale Debtors
Debtors
turnover
Debtors availing discount 4,585 382 30 days
Debtors paying within 40 days 4,012 446 40 days
Other debtors 2,866 509 64 days
11,463 1,337 42 days
Effect of change in creditors payment policy
Creditors turnover in days as given in the question 40.00
Cost of materials (13,770 2 3) for the year 9,180
Creditors outstanding (9,180 40 360) 1,020
Revised creditors with turnover of 60 days (1,020 60 40) 1,530.00
Increase in creditors / release of funds (1,530 1020) 510.00
Saving from funds released (15% 510) 76.50
Discount lost due to delayed payment to creditors (9,180 1%) 91.80
Loss due to delayed payment to creditors (15.30)

Delayed payments to creditors is not feasible -


http://gcaofficial.org
MANAGEMENT ACCOUNTING
SuggestedAnswers
FinalExaminationSummer2013




Page2of6
A.2 Job Costing
Order quantity 2,000,000
Finished running meters (0.04 2,000,000) 80,000
Lamination film input in meters (80,000 98%) 81,633
Printing film input in meters (81,633 97%) 84,158
Set-up consumption 1,200
85,358

Meters Kg Rate Amount in Rs.
Raw materials
Printing film 85,358 * 2,439 260 634,140
Lamination film 81,633 **2,332 260 606,320
Ink (2 84,158 1,000) 168 180 30,240
Chemical (4 84,158 1,000) 337 150 50,550
Glue (81,633 250) 327 110 35,970
1,357,220
Labour cost
Skilled labour [250 (200 90%) 35,000] 48,611
Unskilled labour [750 (200 85%) 15,000] 66,176
114,788
Overheads
Printing overhead (85,358 5) 426,790
Lamination overhead (81,633 3) 244,899
671,689
Total cost 2,143,697
Profit margin (25% on cost) 535,924
Selling price 2,679,621

*85,358 35 = 2,439 **81,633 35 = 2,332


A.3 --------------------------------------Amount in thousands-------------------------------------
Scenarios 1 2 3 4 5 6
Demand 7,000 7,000 8,000 8,000 9,000 9,000
CM per unit 12.00 11.00 12.00 11.00 12.00 11.00
Contribution Margin 84,000 77,000 96,000 88,000 108,000 99,000
Fixed cost other than dep. (15,000) (15,000) (15,000) (15,000) (15,000) (15,000)
Depreciation (300 30)15 (18,000) (18,000) (18,000) (18,000) (18,000) (18,000)
Financial charges (W-1) (25,200) (25,200) (25,668) (25,668) (26,064) (26,064)
Net profit 25,800 18,800 37,332 29,332 48,936 39,936

Equity 140,000 140,000 142,600 142,600 144,800 144,800
Rate of return-% 18.43 13.43 26.180 20.570 33.800 27.580

Probability (Demand) 0.50 0.50 0.30 0.30 0.20 0.20
Probability (CM / unit) 0.35 0.65 0.35 0.65 0.35 0.65
Probability of scenario 0.175 0.325 0.105 0.195 0.070 0.130
Estimated rate of return-% 3.23 4.36 2.749 4.011 2.366 3.585

Probability for getting required rate of return of 20% 0.50

Rate of return based on all the possible scenarios - % cumulative 20.30

W-1: Financial charges
Capital investment 300,000 300,000 300,000 300,000 300,000 300,000
Working capital 50,000 50,000 56,500 56,500 62,000 62,000
350,000 350,000 356,500 356,500 362,000 362,000
Equity 40% 140,000 140,000 142,600 142,600 144,800 144,800
Bank borrowing 210,000 210,000 213,900 213,900 217,200 217,200
Finance charges @ 12% 25,200 25,200 25,668 25,668 26,064 26,064
MANAGEMENT ACCOUNTING
SuggestedAnswers
FinalExaminationSummer2013




Page3of6
A.4 (a) Fixed cost of B: Rupees
Decrease in variable and fixed costs (2,500,000 1,800,000) 700,000
Decrease in units of product B (10,000 2,000) 8,000
Variable cost per unit (700,000 8,000) 87.50
VC for 2,000 units @ 87.50 per unit 175,000
Fixed cost of B (1,800,000 175,000 ) 1,625,000

Contribution margin (CM) per unit of B (based on 2013 results)
Sales 750.00
Raw material (300 + 15%) 345.00
Direct wages 200.00
Variable overheads 87.50
632.50
CM per unit of B 117.50

Shut down point:
Avoidable FC / CM per unit
Avoidable FC (1,625,000 162,500 450,000) 1,012,500
Units to breakeven i.e. shut down point (1012500 117.50) 8,617

(b) Contribution Margin (CM) per unit of A :
Sales 2000
Raw material (500 + 15%) 575
Direct wages 375
Variable overheads W-1 50
1000
1000

Net profit of A for 31.03.13 (20 5 3.75 3) 8,250,000
Net loss of B for 31.03.13 (1.5 6 4 1.8) (1,300,000)
Profit for 2013 6,950,000
Increase in profit required (20% of 6,950,000) 1,390,000
Fixed cost A W-1 2,750,000
Fixed cost B (162,500 + 450,000) (plant is shut down) 612,500
Variable cost of 13,000 units at Rs. 1,000 per unit 13,000,000
Contribution Required 24,702,500

No. of units to be produced 13,000

Price per unit of A to be charged (24,702,500 13,000) 1900

W-1:
Fixed cost of A 2014 2013
Total variable and fixed cost of 13,000 and 10,000 units of A 3,400,000 3,000,000
Increase in fixed cost during 2014 250,000
3,150,000 3,000,000
Variable cost of 3,000 units 150,000
Variable cost per unit (150,000 3,000) 50
VC at 13,000 and 10,000 units 650,000 500,000
Fixed cost of A 2,750,000 2,500,000



MANAGEMENT ACCOUNTING
SuggestedAnswers
FinalExaminationSummer2013




Page4of6
A.5



Activity
Expected
Time
ES EF LS LF Float
1 2 5 0 5 0 5 0
1 3 9 0 9 6 15 6
1 4 8 0 8 6 14 6
2 4 6 5 11 8 14 3
2 5 5 5 10 5 10 0
3 6 9 9 18 15 24 6
4 6 10 11 21 14 24 3
5 7 10 10 20 10 20 0
6 8 7 21 28 24 31 3
7 8 11 20 31 20 31 0

S. No. Path Duration in hours
(i) 1 - 2 - 5 - 7 - 8 31
(ii) 1 - 2- 4 - 6 - 8 28
(iii) 1 - 3 - 6 - 8 25
(iv) 1 - 4 - 6 - 8 25


Therefore, the critical path is path (i)














MANAGEMENT ACCOUNTING
SuggestedAnswers
FinalExaminationSummer2013




Page5of6
A.6 (a) (i) Net profit for the year South East
Sales (24,000 900), (26,400 1,100) 21,600,000 29,040,000
VC (24,000 700), (26,400 775) (16,800,000) (20,460,000)
Contribution margin for the year 4,800,000 8,580,000
FC (24,000 100), (26,400 150) (2,400,000) (3,960,000)
Net profit for the year 2,400,000 4,620,000

Residual Income for the year
Net profit 2,400,000 4,620,000
Required return on assets [15% (11.10)] [15% (25.8)] 1,665,000 3,870,000
Residual Income for the year 735,000 750,000

Return on investment (ROI)
Net profit 2,400,000 4,620,000
Average operating assets 11,100,000 25,800,000
ROI % [2,400,000 11,100,000] [25,800,000 4,620,000] 21.62% 17.91%

RI reveals the same level of performance for each division.
ROI indicates that division East is the underperforming division. However,
one reason could be that South has older plant and machinery and East has
been established relatively recently and therefore the ROI of South is higher
due to the fact that its plant may have been substantially written down due to
depreciation.
To make a better comparison, a more appropriate base should be used. These
may include fair value of assets or historical cost duly adjusted for rate of
inflation. However, in case the base is changed, due recognition should be
given to the fact that older machinery is less efficient and requires more
maintenance etc.

(ii) Units that should be sold to achieve ROI as that of South division
Required return 21.62%
Average operating assets 25,800,000
Amount of return 5,577,960
Fixed costs per annum 3,960,000
Total contribution required 9,537,960

Contribution margin per unit: Amount in Rs.
Sale price 1,100
Variable costs (375 + 225 + 175) 775
Contribution margin per unit: 325

Units required to be sold per annum (9,537,960 325) 29,348

(b) Additional contribution (12.5 8,580,000) 1,072,500
Depreciation (4,000,000 400,000) 5 (720,000)
Revised total annual profit (4,620,000 + 1,072,500 720,000) 4,972,500
Operating investment (25,800,000 + 4,000,000) 29,800,000
ROI 16.69%

Revised total annual contribution 4,972,500
Return on assets (15% 29,800,000) 4,470,000
Residual income 502,500




MANAGEMENT ACCOUNTING
SuggestedAnswers
FinalExaminationSummer2013




Page6of6
A.7 Rupees
(a) Actual quantity at actual prices (given) 173,280
(b) Actual quantity in actual mix at standard prices
P 1,680 40 67,200
Q 1,650 30 49,500
R 870 60 52,200
168,900

(c) Actual quantity in standard mix at standard prices
St. Mix St. Price
P 4,200 1535 1,800 40 72,000
Q 4,200 1235 1,440 30 43,200
R 4,200 835 960 60 57,600
172,800
(i) Material price variance (a) - (b) 4,380 Adverse

(ii) Material usage variance
(1) Standard cost of actual output
[3,648 (1,440 32)] 164,160
(2) Actual quantity at standard price 168,900
(3) Difference in above 4,740 Adverse


(iii) Mix variance (c) - (b) 3,900 Favourable


(iv) Yield variance kg
(1) Actual yield 3,648
(2) Standard yield for actual input
(4,200 32) 35

3,840
(3) Difference in above at standard cost
per unit of output [192 (1,440 32)] 8,640 Adverse

(THE END)
MANAGEMENT ACCOUNTING
Suggested Answers
Final Examination - Winter 2012



Page 1 of 6
A.1 SGL Limited

Projected cash-flow statement for the year ending 31 December 2013




Rs. in million

Inflows
Cash from customers W.1 1,328.36
Outflows

Payments for purchases W.3 (318.11)
Payments for expenses W.4 (681.35)
Payments for financial charges 16/4+(16*1.1*3/4) (17.20)
Payments for dividend (80*20%)+(80*1.1*15%) (29.20)


(1,045.86)
Net inflows

282.50

Workings:
W.1: Cash from customers:
Gross sales for 2013 (W.2) 901.25*1.5 1,351.88
Cash sales net of 2% discount 1351.88*20% 270.38
Collection from trade debtors:

Trade debtors: Opening Balance

90.00
Credit sales for 2013 1351.88-(270.38/0.98) 1,075.98
Trade debtors: Closing Balance 90*1.2 (108.00)


1,057.98


1,328.36


W.2: Cost of sales for 2013
Raw material consumption 722*35%*1.2*1.08 327.50
Variable conversion cost 722*45%*1.2*1.1 428.87
Fixed conversion cost (722*20%-3)*1.06+3 152.88
Cost of goods manufactured

909.25
Opening finished goods inventory

89.00
Closing finished goods inventory

(97.00)
Cost of sales

901.25


W.3: Payments for purchases
Raw material consumption W.2 327.50
Raw material - opening inventories (722*35%)*30/360 (21.06)
Raw material - closing inventories 327.5*30/360 27.29
Total purchases for 2013

333.73
Trade creditors - opening balance

40.00

Trade creditors - closing balance 333.73*60/360 (55.62)


318.11


W.4: Payments for expenses
Variable and fixed conversion costs 428.87+152.88-3 578.75
Variable operating cost (100-9-16)*1.2*1.1 99.00
Fixed operating costs (advertisement) 16*1.06 16.96
Total costs for 2013 (excluding depreciation)

694.71
Payables for expenses Opening Balance (722*65%-3+100-9)*35/360 54.18
Payable for expenses Closing Balance 694.71*35/360 (67.54)


681.35



MANAGEMENT ACCOUNTING
Suggested Answers
Final Examination - Winter 2012



Page 2 of 6
A.2 (a) Learning curve applications
Learning curves may assist the management in the following areas.

1 Pricing decisions



Application of a learning technique may assist the management in determining the
cost of a project/order more accurately and thereby quoting the right price especially
where bidding is expected to be highly competitive.
2 Work scheduling



Learning curves enable management to predict the labour requirement more
effectively and to prepare more accurate delivery schedules.
3 Standard setting



Learning curves may assist management in setting of more accurate budgets and
standards.

(b) Cost of producing ordered bodies of washing machines

Material
First 16 batches (16*66,000) 1,056,000

Next 84 batches 84*(66,000/1.1*1.06) 5,342,400
Direct labour cost
Normal hours at Rs. 220 1,760,000
Overtime hours at Rs. 330 686,070
Overheads
Normal hours at Rs. 150 1,200,000
Overtime hours at Rs. 187.5 389,813
Total costs Rs. 10,434,283
Order price at a margin of 25% of the selling price Rs. 13,912,377


W.1: Learning curve %:


Batch No. Cumulative hours
Average hours per
batch
Learning
curve %
1 200.00 200.00
2 (200+160) 360.00 180.00 (180/200) 90%
4 (360+148+140) 648.00 162.00 (162/180) 90%

W.2: Overtime hours:
Learning effect at 90% learning curve -0.152
Hours for first 64 batches 64*200*(64) 6,803
-0.152

Hours for first 63 batches 63*200*(63) ( 6,712)
-0.152

Hours per batch after batch 91
Hours required:
First 64 batches 6,803
Last 36 batches (9136) 3,276
Total hours 10,079
Overtime hours (10,079 8,000) 2,079









MANAGEMENT ACCOUNTING
Suggested Answers
Final Examination - Winter 2012



Page 3 of 6

A.3 RCL



X Y Z Total

Production Units A 50,000 40,000 25,000

Cost allocation by activity:

1. Procurement department

Direct material cost per units at
Rs. 200 per kg. B

400

300

500

Raw material consumption - kg B/200*A C 100,000 60,000 62,500

EOQ - kg

D 10,000 12,000 6,250

No. of purchase orders C/D 10 5 10 25

Procurement department cost

E 1,000 500 1,000 2,500

2. Batch set up cost



Batch size

F 500 250 250

No. of batches A/F G 100 160 100 360

Batch set-up costs

H 1,000 1,600 1,000 3,600

3. Quality control department



Inspection hours per batch

J 20 15 18

Inspection hours G*J K 2,000 2,400 1,800 6,200

Quality department costs

L 1,455 1,746 1,309 4,510

4. Other overheads



Direct labour cost per unit

M 300 350 250

Direct labour hour per unit M/50* N 6 7 5

Total direct labour hours A*N O 300,000 280,000 125,000 705,000

Utilities 1,800 1,680 750 4,230

Salaries of supervisors and foreman 1,500 1,400 625 3,525

Salaries of cleaners and
maintenance staff

600 560 250 1,410

Miscellaneous expenses 300 280 125 705

P 4,200 3,920 1,750 9,870

Total costs Rs. (E+H+L+P) R 7,655 7,766 5,059 20,480









Costs per unit using ABC:

Direct material 400.00 300.00 500.00

Direct labour 300.00 350.00 250.00

Factory overheads R/A 153.10 194.15 202.36

Total

Rs. 853.10 844.15 952.36




















MANAGEMENT ACCOUNTING
Suggested Answers
Final Examination - Winter 2012



Page 4 of 6

A.4 Industrial Tools Limited

Relevant costs of producing Zee
Material
cost Internal production of Beta (W.2) 7,520*757.50

5,696,400


External buying of Alpha (W.2) 13,984*1,100

15,382,400
Direct labour cost

W.4 1,120,000
Variable overheads (W.4) 20,600*120

2,472,000
Total relevant cost

Rs. 24,670,800
Zee selling price at 30% above the relevant costs 24,670,800*1.3 Rs. 32,072,040



W.1: Decision to produce Beta internally or not
Beta - internal production costs per kg
- Variable 520+(50*1.25)+(120*1.25) 732.50
- Fixed (existing) (Not relevant) -
- Fixed (additional) 188,000/9,400*1.25 25.00
757.50
Cost of Beta for each unit of Zee 757.5*2.5 1,894.00
Cost of material Alpha for each unit of Zee 1,100*2 2,200.00
Saving on producing Beta internally 306.00
Hence it is beneficial to produce Beta internally.



W.2: Internal production capacity for the substitute material Beta and buying of Alpha externally

Total hours available

30,000

Hours required for production of Zee

W.4 20,600

Capacity available for production of Beta

Hrs. 9,400

Beta production from the available capacity 9,400/1.25 Kg 7,520

Quantity of Alpha to be purchased externally (10,000-(7,520/2.5))*2 Kg 13,984


W.3: Variable overhead rate per labour hour for Zee and Beta

Variable overhead rate per labour hour 150-(900,000/30,000) 120.00


W.4: Direct Labour Cost for Zee

Units
Hours per
unit
Hours
Wages at higher of Rs. 100 per hour and
Rs. 210 per unit

5,000 2.2 11,000 100 per hour 1,100,000

3,000 2.0 6,000 210 per unit 630,000

2,000 1.8 3,600 210 per unit 420,000

10,000

20,600

2,150,000

Payment of idle hours at 50% (non relevant cost) 20,600 *100/2 (1,030,000)

Relevant labour cost for Zee
1,120,000














MANAGEMENT ACCOUNTING
Suggested Answers
Final Examination - Winter 2012



Page 5 of 6

A.5 ICL

(a) Maximisation of profit using revised policy Division A
(Gamma)
Division B
(Gamma-plus)


--------------- Rupees ----------------
Contribution margin W.1 18,600,000 23,292,500

Fixed cost (7,000,000) (6,000,000)

Profit before Division managers' bonus 11,600,000 17,292,500

Bonus to division managers at 15% of profit after bonus (A) (1,513,043) (2,255,543)

Estimated profit - using revised policy 10,086,957 15,036,957



W.1: Determination of optimal option based on actual cost

Gamma (Division A) Gamma-plus (Division B)

Selling
price
per kg
CM per kg
(Sale-189
W.3)
Demand
(kg)
Total CM
(Rs.)
Selling
price
per kg
CM per kg
(Sale-627.25
W.3)
Demand
(kg)
Total CM
(Rs.)

300 111 150,000 16,650,000 960 332.75 70,000 23,292,500

375 186 100,000 18,600,000 1,080 452.75 50,000 22,637,500

450 261 50,000 13,050,000 1,200 572.75 30,000 17,182,500



W.2: Determination of the optimal option based on transfer price:

Gamma-plus (Division B)

Selling price per kg
CM per kg
(Sale-671 W.3)
Demand (kg) Total CM (Rs.)

960 289 70,000 20,230,000

1,080 409 50,000 20,450,000

1,200 529 30,000 15,870,000


W.3 Actual variable costs/transfer price
Division A
Gamma
Division B
Gamma Plus

----------Rupees----------

Inter-transfer cost from Division A (102+73)/2 - 87.5

Raw material (B) 637.50/2 102.00 318.75

Conversion costs - Variable and fixed 108.00 230.00

Conversion costs - Fixed (7/0.2), (6/0.25) (35.00) (24.00)

Conversion costs - Variable 73.00 206.00

Selling expenses - Variable 14.00 15.00

Total variable cost - Actual 189.00 627.25

Margin of profit to Division A on internal transfers
(W-4) 87.5/2 43.75

Cost based on transfer price 671.00


(b) Profit using transfer price:


CM from external sale W.1 & W.2 18,600,000 20,450,000

CM from internal sale (50,000/2*87.5) 2,187,500 -

Total CM 20,787,500 20,450,000

Fixed costs (7,000,000) (6,000,000)

Profit before Division managers' bonus 13,787,500 14,450,000

Bonus to division managers at 15% of profit after bonus (B) (1,798,370) (1,884,783)

Estimated profit - using transfer price 11,989,130 12,565,217



Increase/(decrease) in bonus on revision of policy (A-B) (285,327) 370,760


W.4: CM per unit on internal transfer of Gamma to Division B:
Maximum transfer price of Gamma (102+108)*1.25 262.50
Variable cost 102+73 175.00
Division A CM from internal transfers 87.50
MANAGEMENT ACCOUNTING
Suggested Answers
Final Examination - Winter 2012



Page 6 of 6

A.6
(a) Fixed overheads variances calculation under absorption and marginal costing:
Absorption costing Marginal costing
1 Fixed overheads are allocated to
production at a standard fixed rate by using
budgeted production.
Fixed overheads are treated as period cost
and charged to profit and loss account.
2 Difference of actual and budgeted
production results in under/over recovery
of fixed overheads and a volume variance.
As fixed overheads are not allocated to
production, volume variance does not
arise.
3 Two variances are worked out i.e.
expenditure and volume variances.
Only one variance is worked out i.e.
expenditure variance.



(b) Ancient Pharma Limited
Profit statement for the month of November 2012


Standard costing using absorption:


Working Units Rupees
Sales - Export J (5,400*20%)*7,000*1.15 1,080 8,694,000


- Local, corporate

(5,400*25%)*7,000*90% 1,350 8,505,000


- Local, others

(5,400*55%)*7,000 2,970 20,790,000


5,400 37,989,000
Standard cost of sales

5,400*(4,000+800+500) (28,620,000)
Gross profit 9,369,000
Favorable/(Adverse) variances:
Material purchase price A 50,000*(486-500) 700,000
Material usage G,B (6,050*8)-49,000)*500 (300,000)
Labour rate C 60,000*(80-88) (480,000)
Labour efficiency H,C [(6,060*10)-60,000]*80 48,000
Variable overhead expenditure C,D,E [60,000*(50-20)-(3,500,000-1,213,250)] (486,750)
Variable overhead efficiency H,C (6060*10-60,000)*(50-20) 18,000
Fixed overhead expenditure D,E (57,500*20)-1,213,250 (63,250)
Fixed overhead volume variance H,D (6,060*10-57,500)*20 62,000


(502,000)
Gross profit after variances 8,867,000

Workings:
A Actual purchased quantity 24,300,000/ 486 kg 50,000
B Actual material usage 4,000+50,000-5,000 kg 49,000
C Actual labour hours used 5,280,000/88 Hours 60,000
D Standard fixed overhead rate (1,150,000/57,500) Rs./hr 20
E Actual fixed overheads (1,150,000*25%*1.10)+(1,150,000*75%*1.04) 1,213,250
F Finished goods production Qty (100+6,300)-(150+250) Units 6,000


Equivalent units:
G - Material 6,000-100+150 Units 6,050
H - Conversion costs 6,000-(100*60%) + (150*80%) Units 6,060
J Sales quantity 6,000+200-800 Units 5,400


(THE END)
MANAGEMENT ACCOUNTING
Suggested Answers
Final Examination Summer 2012

Page 1 of 6
A.1 (a) Himalaya Chemicals Limited





Introduction of the new product WYE


Selling price per unit 190.00 200.00 210.00

Direct material cost per unit (30.00)


Variable operating cost for dept. A and B W.1 (121.75)




(151.75) (151.75) (151.75)

Contribution margin per unit (A) Rs. 38.25 48.25 58.25




Demand in units (B) 18,000 15,000 12,000


Production at lower of demand or capacity of
16,800 units (12,600/0.75) (C) 16,800 15,000 12,000




Total contribution margin (AC) Rs. 642,600 723,750 699,000



Renting out of spare operating facilities Dep't. A Dep't. B

Renting of spare operating hours (42,00030%)

12,600.00 12,600.00

Rental income per hour

140.00 100.00

Variable operating cost per hour W.1 (111.00) (77.00)

Contribution margin per hour

29.00 23.00

Total contribution margin (12,600 (29+23))

Rs. 655,200






HCL should produce 15,000 units of WYE having selling price of Rs. 200 per units which gives the
highest CM to the company.



W.1 Variable operating costs - WYE Dep't. A Dep't. B

Total overheads per hour 145.00 105.00


Fixed overheads per hour {1,356,600/ (42,00095%)},
{1,117,200/ (42,00095%)} (34.00) (28.00)

Variable operating cost per hour 111.00 77.00


Department-wise cost per unit (111*0.75), (77*0.5)
83.25 38.50




Total variable cost per unit

Rs. 121.75




(b) Other matters to be considered in deciding between the two options:




(i) Risk



Introduction of a new product WYE involves a risk that it may not be able to capture the
market at the projected selling price.





(ii) Interference and difficulties with the tenant



In case of renting out of the facilities, the management may have to face day to day disputes
and interferences.




(iii) Timely discontinuation of the contract



HCL may not be able to discontinue the arrangement in time when the facilities are needed for
its own use.

































MANAGEMENT ACCOUNTING
Suggested Answers
Final Examination Summer 2012

Page 2 of 6
A.2 Quality Appliances Limited


Material (kg) Labour hours Machine hours

Kg / Hours per unit HX (2,000/400) 5.00 (960/200) 4.80 (1,000/500) 2.00

HY (2,000/400) 5.00 (650/200) 3.25 (1,500/500) 3.00

Resources available 2,700 2,000 1,340


Objective function:

Maximise Contribution =
*
1,540HX+
**

1,050HY subject to the constraints as mentioned below.
*

HX contribution margin: 6,000-2,000-960-1,000-(62580%)=1,540
**

HY contribution margin: 5,500-2,000-650-1,500-(37580%)=1,050


Constraints:

Material 5HX+5HY 2,700 Eq. 1

Labour hours 4.8HX+3.25HY 2,000 Eq. 2

Machine hours 2HX+3HY 1,340 Eq. 3

HX, HY 100 Eq. 4

Plotting of constraints on the graph:








Number of units at feasible points A, B and C of the graph are worked out as below:


Point A


Putting the value of 100 for HX in equation 3, we get (2100) + 3HY 1,340



HY = 380


Point B

Multiplying equation 3 by 2.4, we get 4.8HX+7.2HY=3,216 Eq. 5

Subtracting equation 2 from Equation 5 we get 3.95HY=1,216

HY = 308

Putting the value of HY in equation 5, we get 4.8HX+2,218=3,216

HX = 208



MANAGEMENT ACCOUNTING
Suggested Answers
Final Examination Summer 2012

Page 3 of 6

Point C


Putting the value of 100 for HY in equation 2, we get 4.8HX+(3.25100) 2,000


HX =349



CM at feasible points A, B and C
HX (Units) HY (Units)
CM
(at Rs. 1,540 and Rs.
1,050 per unit)

A Including minimum required 100 units 100 380 553,000

B Including minimum required 100 units 208 308 643,720

C Including minimum required 100 units 349 100 642,460



QAL should produce 208 units of HX and 308 units of HY to maximise the contribution.


A.3 Spicy Foods Limited



BX BY BZ TOTAL

Units sold (in millions) A

0.400 0.600 0.300 1.300

Labour hours per unit B

1.50 1.75 2.00

Labour hours (in millions) C AB 0.600 1.050 0.600 2.250



Required CM from sale of Jumbo pack to earn a profit of Rs 5 million

Rupees in million

Present sales D

140.000 180.00 126.00 446,000

Variable cost



Total cost of sales E

105.000 135.000 120.000 360.000

Fixed cost of sales

135/2.25C (36.000) (63.000) (36.000) (135.000)

Variable cost of sales F

69.000 72.000 84.000 225.000

VC of sales reduced by 2%

F98% 67.620 70.560 82.320 220.500

Variable operating costs G A45, 49, 26 18.000 29.400 7.800 55.200

Revised total variable costs H

85.620 99.960 90.120 275.700

CM from present sales J D-H 170.300

CM from reduced 80% sales

(170.300/80%) 136.240

Fixed COS and operating costs 135+(30+49+13-55.2) (171.800)

Additional fixed cost (3+4) (7.000)

Required profit for the year (5.000)

(47.560)

Number of Jumbo packs to earn a profit of Rs. 5.0 million:


Sales price per Jumbo pack (350+300+420)90% 963.00

VC of sales per Jumbo pack H/A 214.05 166.60 300.40 (681.05)

CM per unit per Jumbo pack Rs. 281.95

Number of Jumbo packs to be sold ( 47,560,000/281.95) Units 168,682
















MANAGEMENT ACCOUNTING
Suggested Answers
Final Examination Summer 2012

Page 4 of 6
A.4 Sky Limited
(a)



GAMMA
(New facility)
ALPHA (Existing facility)


Option 1 Option 2 Option 3

Selling price per unit A 970.00 650.00 700.00 750.00

Variable cost per unit B W.1 (651.25) (485.00) (485.00) (485.00)

CM per unit
C A-B 318.75 165.00 215.00 265.00

Hours per unit (existing
facility) D 3.50* 2.00 2.00 2.00

CM per hour (Rs.) E C/D 91.07 82.50 107.50 132.50

Demand in units
F

100,000 200,000 160,000 120,000

Production capacity:



Hours G
144,000 440,000 440,000 440,000

Units H
(G/2.4)
60,000
G/2.0
220,000
G/2.0
220,000
(G/2.0
220,000

Demand in units
J

100,000 200,000 160,000 120,000

Product having higher CM per hour will be produced
first and restricted to lower of capacity / demand - units K
Note 1
Beta
60,000
Alpha
160,000
Alpha
120,000

Remaining capacity will be used for the products
having lower CM per hour - units L
Alpha
(G-(K*3.5)/2
115,000
Beta
(G-(K*2)/3.5
34,286
Beta
(G-(K*2)/3.5
57,143

Contribution margin; Alpha
M
L*265
(Note.2)
30,475,000
K*C
34,400,000
K*C
31,800,000

Contribution margin; Gamma (at Rs. 318.75 per unit) N 19,125,000 10,929,663 18,214,331

Total contribution margin

49,600,000 45,329,663 50,014,331


*Hours for Beta only (conversion from Beta to Gamma would take place in the new facility)

Conclusion

: SL should use option # 3 for producing 120,000 units of Alpha and 57,143 units of Beta and
Gamma to maximise the profit.


Note 1 In case of option # 1 production of Beta / Gamma would be restricted to 60,000 units which is
the maximum capacity of the proposed new facility for producing Gamma.

Note 2 Alpha would be produced by utilising the capacity remained after producing Beta. As a result,
production of Alpha will be restricted to 115,000 units and the sale price of Rs. 750 will be
applicable as per option # 3 resulting in CM of Rs. 265 per unit.






W-1




Per unit variable cost of Alpha (590-(23,100,000/440,0002) 485.00
Per unit variable cost of Gamma:
Var. cost of Beta in the existing facility (735-23,100,000/440,0003.5) 551.25
Var. cost of Gamma in the new facility (300-(12,000,000/144,0002.4) 100.00
Var. cost of producing Gamma 651.25

(b) Transfer price of Beta

Before establishment of the proposed facility for Gamma,
maximum contribution margin can be obtained from Alpha
Option # 2
160,000215 34,400,000

After establishment of the proposed facility for Gamma,
maximum contribution margin can be obtained from Alpha
Option # 3
120,000265 31,800,000

Loss of CM due to decrease in production of Alpha 2,600,000

Loss of CM per unit of Beta
2,600,000 / 57,143
45.50

Variable cost per unit of beta in the existing facility W -1 551.25

Minimum transfer price per unit for Beta 596.75





MANAGEMENT ACCOUNTING
Suggested Answers
Final Examination Summer 2012

Page 5 of 6
A.5 Super Autos

LV MV

----- Rupees -----
Cost per unit under Activity Based Costing:

Direct raw material 850,000/10,000; 900,000/12,000 85.00 75.00
Direct wages (3,000,000/30,000) 1,000/10,000 10.00


(3,000,000/30,000) 1,500/12,000

12.50
Overheads 798,750/10,000; 715,883/12,000 79.88 59.66


174.88 147.16


Cost per unit under the single factory overhead rate based on direct labour hours:
Direct raw material

85.00 75.00
Direct wages

10.00 12.50
Overheads *(8,600,000/30,000) 1,000/10,000 28.67


*(8,600,000/30,000) 1,500/12,000

35.83

123.67 123.33
*(2,500,000+1,500,000+2,000,000+1,000,000+1,600,000) = 8,600,000


W.1 Overheads allocation to LV and MV based on ABC method: LV MV

Production; 97A 485,000 349200

Procurement 22,750B 113,750 136,500

Finished goods stores and dispatch 16,458.338, 10 i.e. No. of sales orders 131,667 164,583

Quality control 1,366.67C 68,333 65,600

798,750 715,883



LV MV
W.2 Machine hours A (25200)= 5,000 (24150) = 3,600

No. of purchase orders B (850,000/170,000)= 5 (900,000/150,000)= 6

No. of quality inspections C (10,000/4002)= 50 (12,000/5002)= 48


W.3 Computation of cost driver rate: Production Procurement
F. goods
stores &
dispatch
Quality
control
Machines operating expenses 2,500,000 - - -
Maintenance expenses;
1,500,00070%, 5%,15%, 10%

1,050,000

75,000

225,000

150,000
Technical staff expenses:




- Maintenance; (2,000,00050%)70%,
5%,15%, 10%

700,000

50,000

150,000

100,000
- Others; 2,000,00030%, 12%, 0%, 8% 600,000 240,000 - 160,000
Procurement - 1,000,000 - -
Finished goods stores & dispatch - - 1,600,000 -
Overheads by department 4,850,000 1,365,000 1,975,000 410,000

Cost drivers


50,000
machine
hours
60 Purchase
orders
120 Sales
orders
300 (1502)
Quality
inspections
Cost per driver 97 22,750 16,458.33 1,366.67














MANAGEMENT ACCOUNTING
Suggested Answers
Final Examination Summer 2012

Page 6 of 6
A. 6 Zen Trading Limited
(a)


Present cost of recoveries:



Interest cost

Delay in collection Amounts collected Interest at 16%
Rs. in
million

1 Month 26%10099% 25.740 0.343


2 Months 34%100 34.000 0.907


3 Months 30%100 30.000 1.200


5 Months 5%100 5.000 0.333


11 Months 4%100 80% 3.200 0.469 3.252

Discount cost for payment in one month 261% 0.260

Credit Control Department cost 1.2/12 0.100

Recovery cost of Credit Control Department 1005%10% 0.500

Immaterial balances written off 1001% 1.000

Retention fee for the legal firm

0.025

Recovery fee of the legal firm 1004%80%20% 0.640

Balances written off after legal proceedings 1004%20% 0.800

6.572

Cost of recoveries under factoring:


Factoring service charges at 5% 1005% 5.000

Interest charges for recovery in 15 days 10095%16%0.5/12 0.633

5.633

Savings on accepting factoring proposal 0.944



Conclusion:
ZTL should accept the proposal of the factoring company as it will result in a saving of Rs.
944,000.


(b) Difficulties ZTL may have to encounter after accepting factoring arrangement.
(i) There may be disagreements between the factor and the company, over credit evaluation of
the customer.

(ii) A customer may feel uncomfortable in dealing with a factor.

(iii) The factoring of trade debts may be considered as a symptom of weakness of the
management.




The above difficulties can be resolved by:

(i) Incorporating appropriate clauses in the agreement with the factor to avoid disagreements
over credit evaluation of the customers.



(ii) Keeping continuous personal contacts with customers and resolving their problems
promptly.



(iii) Effective public relationing with all the stakeholders.


(THE END)

MANAGEMENTACCOUNTING
SuggestedAnswers
FinalExaminationWinter2011

Page 1 of 6

A.1 Relevant costs for the tender
Working
s Rs. in 000
Existing capacity's CM lost 1 6,913
Material A (8,000 Rs. 820) 6,560
Material B (500 Rs. 600) 300
Material B - loss on sale of redundant stock 2 30
Labour costs for project 3 3495
Factory overhead 4 2,752
Cost of machine 5 1,140
21,190
Profit margin on cost (20%) 4,238
Bidding price 25,428

W-1: Contribution margin lost
Capacity required for order 40%
Present Idle capacity 25%
Existing utilized capacity to be used for new project 15%
Present quarterly contribution margin
[Rs. 350,000 (Sales) - Rs. 234,500 (material) - Rs. 28,650 (labour inclusive of idle
labour) - Rs. 23,000 (variable FOH) - Rs. 12,000(variable selling expenses)] 51,850

Contribution forgone for 60 days (15%/75% Rs. 51,850 2/3) 6,913


W-2: Loss on sale of redundant Material B stock
Kgs
Available stock 6,000
Usage during first 30 days (2,000)
Usage during next 60 days (6,000 2/360/75) (3,200)
Usage for new project (500)
Redundant stock 300

Loss on sale of redundant stock at current sale price [(Rs. 700-600)300) 30


W-3: Labour cost for the project
Skilled Unskilled
Total hours required for project 6,000 15,000
Idle hours available
Idle hours due to curtailment of activity from 75 to 60%
22,500 15/75 2 9,000
36,000 15/75 2 14,400
Present idle hours
22,500 10/90 2 5,000
36,000 4/96 2 3,000
14,000 17,400

Payment for the tender activity Rs. in 000
6,000 200 1,200
3,000 200 80% 480
14,400 125 1,800
600 125 20% 15
1,680 1,815
Total labour cost 3,495




MANAGEMENTACCOUNTING
SuggestedAnswers
FinalExaminationWinter2011

Page 2 of 6


W-4: Variable overheads of the project
Rs. in 000
Variable factory overhead rate 23,000/(58,5003) 0.13105
No. of hours required for the new project 21,000
Variable overhead for two months 2,752

W-5: Cost of machine relevant for the project
Purchased cost of machine 4,500
Less: Resale price or savings in labour costs whichever is higher:

Resale price 3,000

Savings in labour costs
Machine life in # of months 36
Less: Project life 2
Life after the project 34
Labour costs for one months(Rs. 20022,500+ Rs. 125 36,000) 9,000
Savings in 34 months (Rs. 9,000 5% 34) 15300
Less : 80% payable for idle time (12240)
Add: Sale of old machine 300
3,360 3,360
1,140

A.2 (a) Sales volume variance
Products
Total
XA YA ZA
Budgeted sales (units) 60 28 20
Actual sales (units) 80 24 30
Volume variance in units (20) F 4 A (10) F
standard margin per unit

----- Rupees -----

XA : 200,000 - 39,500 - 80,000 - 100,000 (19,500)

YA : 300,000- 54,000 -100,000- 125,000

21,000

ZA : 475,000 - 78,000 -150,000- 187,500



59,500

Volume variance 390,000 A 84,000 A (595,000) F (121,000) F

(b) Sales price - planning variance
Actual sales of XA at budgeted price (Rs. 200,000 80) 16,000,000
Revision in Budget
Promotional Sales (180,000 35) (6,300,000)
Other than Promotional Sales (200,000 45) (9,000,000)
(700,000) A

Sales price - operational variance
Actual sales of XA [Rs.37,425,000 - (Rs.300,000 24) - (Rs. 475,000 30)] 15,975,000
Actual sales of XA at revised budgeted price (6,300 + 9,000 as above) (15,300,000)
675,000 F

(c) Labour efficiency - planning variance
Standard time
XA: 80 (80,000 100) 64,000
YA: 24 (100,000 100 ) 24,000
ZA: 30 (150,000 100) 45,000
133,000
Revision in standard (8% improved performance) [133.000 92%] 122,360
Variance in hours (saved) 10,640
standard rate per hour (Rs. 100 1.05) 105
1,117,200 F

MANAGEMENTACCOUNTING
SuggestedAnswers
FinalExaminationWinter2011

Page 3 of 6

Labour efficiency - operational variance
Standard time (revised) 122,360
Actual time (120,000)
Variance in hours 2,360
standard rate per hour 105
247,800 F


A.3 Jan-12 Feb-12 Mar-12 Apr-12
May-
12 Total
----- Rupees in '000 -----
Receipts
Mobilization advance 35,000 35,000
Running bills (RB) 42,000 63,000 35,000 - 140,000
Less: Mobilization advance (in the ratio of
RB) (10,500) (15,750) (8,750) - (35,000)
Retention money (5% of RB) (2,100) (3,150) (1,750) - (7,000)
29,400 44,100 24,500 98,000
Sales tax @16% of billing amount less
mob adv 5,600 5,040 7,560 4,200 22,400
Withholding tax @ 6% (2,436) (2,066) (3,100) (1,722) (420) (9,744)
Release of retention money - - - - 7,000 7,000
Net receipts 38,164 32,374 48,560 26,978 6,580 152,580
Payment of sales tax (5,097) (4,200) (9,297)
Payment to supplier of equipments (95,000) (95,000)
Installation charges (11,131) (25,972) - (37,103)
27,033 32,374 (77,509) 22,778 6,580 11,256

WORKINGS:
Input/output adjustment of sales tax Jan-12 Feb-12 Mar-12 Apr-12
Output tax as worked out above 5,600 5,040 7,560 4,200
Less: Input adjustment (16/11695,000) (13,103) (7,503) (2,463)
Paid / (excess) carried forward (7,503) (2,463) 5,097 4,200

Cost of installation and related works
Contract price 140,000
Less: Profit margin @15% 21,000
Project cost 119,000
Less: Cost of equipments (100 / 116
95,000) 81,897
Cost of installation and related works 37,103


A.4 STATEMENT OF SAVINGS AND ADDITIONAL COSTS
Workings Rs. in 000
Materials 1 5,329
Labour 1 56,481
Warranty 2 728
Factory overheads - ordering and holding costs 3 13,331
Savings in factory overheads (other than ordering & holding costs & depreciation) 4 5,264
Increase in factory overheads due to increase in depreciation (9,000-2,700) (6,300)
74,833

Conclusion:
Implementation of the consultants' suggestions would increase the gross margin by more than
17% (74,833 434,000 = 17.2%) and therefore consultants' suggestions should be accepted.
However, their suggestion as regards JIT system is not feasible as against the savings of
Rs.13.3 million on inventory ordering and holding costs, discount on bulk purchases
amounting to Rs.14.5 million would be lost.
MANAGEMENTACCOUNTING
SuggestedAnswers
FinalExaminationWinter2011

Page 4 of 6

W-1 : Savings in cost of labour and material on account of purchase of new machine
Material Labour
Cost of labour and material if new machine is NOT purchased Rs. in 000
Current years cost (A) Note: 26,000 units = 3540 + 24960 2500 834,400 138,600
Cost to produce 2000 additional units in next year at current rates
(2,000 units / 26,000 A) 64,185 10,662
Cost to produce 28,000 units at current rates 898,585 149,262
Impact of 8% increment in material and 10% increase in labour costs 71,887 14,926
Cost of labour and material if new machine is NOT purchased 970,472 164,188

Cost of labour and material if new machine is purchased
Cost of material before wastage (970,472 0.96) 931,653
Wastage under new machine (931,653 2 /98) 19,013
Cost of material after discount 950,666
Loss of discount if bulk purchases discontinued (950,6661.5/98.5) 14,477
Skilled labour (164,188 8 /25) 52,540
Unskilled labour (164,188 /25 12 70%) 55,167

Cost of labour and material if new machine is purchased 965,143 107,707

Net savings in labour and material costs 5,329 56,481

W-2 : Savings in cost of warranty Rs.in000
Costs under present conditions (Rs. 4,000 28000 1%) 1,120
Less: Costs if new machine is purchased (Rs. 3,500 28000 0.4%) 392
Net savings 728
W-3: Savings in ordering/holding costs
Existing Proposed
Size of order (B) 14,000 [14,000 (1-85.71%)] 2,000
No. of orders (28,000 B) (C) 2 14
Avg. inventory (B/2+ 4,000) (D) 11,000 (B/2) 1,000
Rs.in000 Rs.in000
Ordering costs (Rs. 45,000 C) 90 630
Holding costs (970,472 D / 28000 4%) 15,250
(965,144 D/28,000 x
4%) 1,379
Total ordering and holding costs 15,340 2,009

Savings in ordering and holding costs (Rs. 15,340 - Rs. 2,009) 13,331

W-4: Factory overheads
Existing
As per annual accounts 193,502
Less: Depreciation (Rs. 54 - Rs. 5.4) /18 (2,700)
Ordering and holding costs as above (15,340)
175,462
Savings in factory overheads {175,462x(10%-7%} (other than depreciation and
inventory ordering and hold costs 5,264







MANAGEMENTACCOUNTING
SuggestedAnswers
FinalExaminationWinter2011

Page 5 of 6

A.5 Process Account - Department 2
Units Rs. Units Rs.
Opening work in process 2,000 128,750 Normal loss account (W-1) 2,500 37,500
Received from Dept 1 53,000 2,057,500 Transferred to Dept 3(W-1) 48,000 3,598,750
Direct Material 988,000 Closing work in process (W-1) 5,000 307,500
Direct wages 488,000
Production overheads 244,000

Abnormal gain (500 75) 500 37,500

55,500 3,943,750 55,500 3,943,750

W-1: Costs computation
Rs.
Cost of units transferred to Department 3
Opening
WIP: Balance as at November 1, 2011 (2,000 units) 128,750
Material (400 [W-2] x Rs. 20 [W-3]) 8,000
Labour & Prod OH
(800 [W-2] x Rs. 15 [W-
3]) 12,000
148,750
Introduced and completed within the month (46,000 x Rs. 75) 3,450,000
3,598,750

Cost of closing work in process
Transferred from Department 1 (5000 x Rs. 40 ) 200,000
Material
(3,500 [W-2]xRs.20 [W-
3]) 70,000
Labour & Prod OH
(2,500 [W-2]xRs.15 [W-
3]) 37,500
307,500

W-2 : Equivalent Production Unit (EPU)
Total
Transferred
from dept 1
Material
introduced
in dept 2
Labour &
Prod. OH
------------ Units ------------
Opening WIP 2,000
Transfer in 53,000
55,000
Accounted for
Opening WIP Completed 2,000 - 400 800
Started and completed 46,000 46,000 46,000 46,000
Closing WIP 5,000 5,000 3,500 2,500
Normal loss [(55,000 - 5,000)0.05) 2,500 - - -
Abnormal gain (2,500 - 2,000) (500) (500) (500) (500)
55,000 50,500 49,400 48,800
W-3 : Cost per unit for each element
Cost (Rs.)
EPU
(W-2)
Cost per
unit (Rs.)
Transfers from dept 1 2,057,500
Less: Scrap value of normal loss (2,500 Rs. 15) 37,500
2,020,000 50,500 40
Material 988,000 49,400 20
Wages 488,000 48,800 10
Overheads 244,000 48,800 5
Total costs per unit 75
MANAGEMENTACCOUNTING
SuggestedAnswers
FinalExaminationWinter2011

Page 6 of 6



A.6


Computation of total hours required for 1st and repeat orders
Rupees
Sale price per unit 10,500
Less: Margin @ 20% 2,100
Cost 8,400
Less: Costs not affected by learning curve (W-1) 4,250
Costs dependent on learning curve A 4,150


Variable cost (labour & overheads) per hourDept B (Rs.
2001.25)
B
250
Avg. labour hours per unit for 1st and repeat order C = A / B 16.60
Labour hours per unit for 1st order D 20
Learning curve factor E = C / D 0.83
Relevant cumulative total volume factor as per table F 1.80
Units for 1st order G 500
Total units for 1st and repeat order H (F G) 900
Repeat order J (H-G) 400


Applied fixed overheads - to be ignored


Working 1
Costs not to be affected by learning curve
Direct Material 3,350
Direct labour in Department A 720
Variable overheads (25% of labour cost in Department A) 180
4,250

(THE END)
MANAGEMENT ACCOUNTING
Suggested Answers
Final Examinations Summer 2011

A.1 (a) Budgeted sales 8,250,000 / 275 units 30,000
Sale price variance 275 10%
(30,000/3x1.2)
Rs. 330,000 A
Actual sales (8,745,000 + 330,000) /
275
units 33,000
Sale volume variance (33,000 30,000) 275 Rs. 825,000 F
Material price variance A (36.4 35) 63,900 2/3 Rs. 59,640 A
B (20.8 20) 105,600 2/3 Rs. 56,320 A
Total material price variance Rs. 115,960 A

Material mix variance
Actual mix at standard cost- Material A 63,900 35 Rs. 2,236,500
- Material B 105,600 20 Rs. 2,112,000
Rs. 4,348,500
Standard mix* (35 2 + 20 3) 169,500 (130* / 5) 4,407,000
Mix variance Rs. 58,500 F

Material yield variance:
Standard output from actual input 169,500 / 5 units 33,900
Actual output units 33,000
Yield variance units 900 s
Yield variance 900 130 Rs. 117,000 A

Labour rate variance
Standard rate per hour 2,700,000 / (30,000 1.5) Rs. 60
Actual average rate per hour (60 + 66 + 66) / 3 Rs. 64
Actual labour hours worked 3,041,920 / 64 hours 47,530

Labour rate variance 47,530 (64-60) Rs. 190,120 A

Labour efficiency variance:
Standard labour hours required 33,000 1.5 49,500
Actual labour hours used 47,530
Efficiency variance hours 1,970 F
Labour efficiency variance 1,970 60 Rs. 118,200 F

(b) (1) Sales price variance:
The variance arose on account of discount allowed by the company in March 2011.

The decision to allow this discount does not seem appropriate on account of the following:
The company had already managed to surpass the budgeted sales quantity in January & February 2011.
The raw material prices and labour rate have seen increased from February, 2011.
After allowing discount, the sale in March made less contribution margin as compared to February.
(2) Yield variance:
Although there could have been more than one reason for the above variance, it may have been due to.
use of cheaper materials as indicated by favourable mix variance.

Another important point is that the adverse yield variance may have been partly on account of poor labour
performance. This aspect needs to be investigated.

(3) Adverse labour rate variance:
It was on account of increase in wages which was a conscious decision of the management. It seems that in
the short run i.e. in February and March the extra cost could not be recouped by the increased efficiency.
However, in the long run it may have far reaching positive effect.


MANAGEMENT ACCOUNTING
Suggested Answers
Final Examinations Summer 2011
Punjnad Juice Company statement for projected working capital:
A.2
No. of Units
Year 1 Year 2
Opening stock A 1,000
Production B 15,000 20,000
Sales C 14,000 18,000
Closing stock D 1,000 3,000
---------------
--
Rupees -------------------
-
Current Assets:
Stock Raw material (B 80 3/12) 300,000 400,000
Finished goods W-1 168,000 469,714
Debtors rate of provision for doubtful debts (200
C/12)X0.9925
231,583 297,750
699,583 1,167,464
Current liabilities:
Creditors for supply of raw material W-2 250,000 283,333
Creditors for expenses W-3 65,833 72,667
315,833 356,000

Working capital required Net 383,750 811,464

W-1 Finished goods

Materials ( B x 80) 1,200,000 1,600,000
Labour & variable overheads (B 40) 600,000 800,000
Fixed overheads (20 24,000) 480,000 480,000
Depreciation (10 x 24,000) 240,000 240,000
Cost of goods manufactured E 2,520,000 3,120,000

Finished goods
{E/B1,000} and {(E+168,000)/(B+A) D} 168,000 469,714

W-2 Creditors for raw material
Material consumed(B 80) 1,200,000 1,600,000
Closing stock (1,200,000 x 3/12) 300,000 400,000
Opening stock - -300,000
Purchases during the year F 1,500,000 1,700,000
Creditors outstanding (F 2/12) 250,000 283,333


W-3 Creditors for expenses
Variable overheads (10 B/12) 12,500 16,667
Fixed overheads (20 24,000/12) 40,000 40,000
Fixed selling expenses (2 24,000/12) 4,000 4,000
Variable selling expenses (8 C/12) 9,333 12,000
65,833 72,667

After option 2
A.3 W-1 Burger Fries Cold drink Ice cream Total
Sale price per unit A 150 50 40 80
Sale ratio B 6 7 8 3
Weightage (A B) C 900 350 320 240 1810
Item wise Sale (Rs. in million) D 90 35 32 24 181
Contribution % E 40% 45% 50% 60%
Contribution Margin (Rs. in million) (D E) F 36.00 15.75 16.00 14.40 82.15

Average contribution margin %
(82.15/181100)

45.39%

Cost per deal{A (100E)}
90.00 27.50 20.00 32.00 137.50
Cost per Deal 2
90.00 27.50 20.00 32.00 169.50



MANAGEMENT ACCOUNTING
Suggested Answers
Final Examinations Summer 2011
OPTION 1 Rupees
Sale of individual items 30% (1.25x181 million) 67,875,000
Sale of deals 70% (1.25x181 million) 158,375,000
Total sale @ 25% (1.25 181 million) 226,250,000

Contribution Sale CM % W-1 30,808,463
Individual items 67,875,000 45.39 W1 32,802,630
Deal 1 (60% of 158,375,000) 95,025,000 34.52 W1 24,997,910
Deal 2 (40% of 158,375,000) 63,350,000 39.46 88,609,003
Total contribution from option I 226,250,000

W 1: Contribution margin on Deals
Deal 1 Deal 2
Selling price 210.00 280.00
Cost of deals 137.50 169.50
Contribution margin 72.50 110.50
Contribution margin % 34.52 39.46

OPTION - 2
Total sale (1.35 181 million) 244,350,000
Home delivery sale 30,000,000
Outlet sales 214,350,000


Sale
price
Sale
ratio
Weight-age
Item wise Sale Contri-
bution
%(W.2)
Contribution
Margin
Outlets Home Delivery Total
Burger 120 6 720 106,582,873 17,197,452 123,780,325 25.00 30,945,081
Fries 40 7 280 41,448,895 6,687,898 48,136,793 31.25 15,042,748
Cold
drink 32 8 256 37,896,133 6,114,650 44,010,783 37.50 16,504,044
Ice
cream 64 3 192 28,422,099 - 28,422,099 50.00 14,211,050
1448 214,350,000 30,000,000 244,350,000 76,702,923

Additional cost on home delivery sets:
Variable cost of home delivery 30,000,000/600*20
Fixed cost of home delivery
Total contribution from option II

(1,000,000)
74,852,923
(850,000)
W-2 COMPUTATION OF REVISED CONTRIBUTION MARGIN PER UNIT

Sale price
(Rs.)
Present cost
(Rs.)
Revised CM (Rs.) CM %
Burger 120 90 30.00 25.00
Fries 40 27.5 12.50 31.25
Cold
drink 32 20 12.00 37.50
Ice
cream 64 32 32.00 50.00
Conclusion: Option 1 is more profitable
A.4 MW Oct-Mar Apr-Jly Aug-Sep Total
W-1 Factory consumption capacity A 4.00
Non-production use B 0.25 0.25 0.25 0.25
Factory consumption @100 cap. C 3.75
Machine / capacity usage D 1/380% 2/3x90% 3/3100%
Factory consumption (CD)
MW
E 1 2.25 3.75
Total consumption F 1.25 2.5 4
Available for sale (5.0 Mw F) G 3.75 2.5 1
Number of DAYS H 180 120 60
Chargeable units to utility 16,200,000 7,200,000 1440,000 24,840,000











MANAGEMENT ACCOUNTING
Suggested Answers
Final Examinations Summer 2011
Computation of total and per megawatt costs:
Rupees
Operational (12*1,500,000) 18,000,000
Labour Cost (12x250,000) 3,000,000
Other related costs (12x500,000) 6,000,000
Depreciation (Rs 100 - 4 million) / 6 years 16,000,000
Fuel Cost (12x24,000,000) 288,000,000
Total generation cost 331,000,000



Financing cost of interconnecting structure (15M*16%) 2,400,000
Profit required 60,000,000
393,400,000
Less: Amount presently being paid 180,000,000
Amount to be charged from utility company 213,400,000
Chargeable units to the utility company to be charged to the utility company 8.59

Landed cost price of good unit
A.5

W-1 FML
Pak
LMN China PQR
Singapore
----------- -- Rupees- -------------
Purchase price per units 287.50 265.00 280.00
Freight-in per unit 2.00 9.00 5.00
C & F value space 289.50 274.00 285.00
Import duty at 10%{(126.50/1.15)-100} - 27.40 28.50
289.50 301.40 313.50
Cost per good unit (289.50 / 0.93) (301.40/0.99)
(313.50/0.99) 311.29 304.44 316.67
Recovery through sale of defective rang material @Rs.4 (40/100)(40/100)
(40x0.0.07) / (40x0.01) per unit (2.80) (0.40) (0.40)
308.49 304.04 316.27
Purchasing priority 2 1 3

W-2 ANNUAL REQUIREMENT OF PRIMARY RAW MATERIAL AND QUANTITIES OUGHT TO BE PURCHASED FROM EACH SUPPLIER

Capacity of Department B net of normal losses
Units
4,000,000
Output loss -(5/95*4,000,000) 210,526
required in department B 4,210,526
Material required to be put into Department-A to get above output (4,210,526/90%) 4,678,362


Totals Defective Good
C&F per
units
Cost
To be purchased from LMN
Rupees
2,000,000 20,000 1,980,000 301.40 602,800,000
To be purchased from FML 1,600,000 112,000 1,488,000 289.50 463,200,000
To be purchased from PQR (Balancing) 1,222,588 12,226 1,210,362 313.50 383,281,338
144,226 4,678,362 1,449,281,338
Proceeds from sale of scrap units (144226*40) (5,769,040)
Cost material 1,443,512,298



Statement of department wise cost
Quantities
Received from preceding dept(4000,000/0.95)
: -------------------No. of units---------------------
4,210,526
Put into department (4,210,526/0.9) 4,678,362
Process losses (467,836) (210,526)
4210,526 4,000,000
Cost uncured: Depart-B Depart-B
Rupees
Raw material/cost from provably dept W-2 1,443,512,298 1,818,635,094
Wages (4,210,526 18/60200);(4,210,526 12/60250) 252,631,560 210,526,300
Variable over heads (60% and 75% of wages) 151,578,936 157,894,725
Fixed over heads (10M
1,263,158/2,105,263)(10M842,105/2,105,263)
6,000,000 4,000,000
Sales of Scrap (75467,836)(125210,526) (35,087,700) (26,315,750)
Total cost of goods manufactured 1,818,635,094 2,164,740,369
Cost per unit 431.93 541.19

A-6
LINEAR PROGRAMMING

Variables

Let x = number of tables

Let y = number of chairs

Constraints
Machine hours x + 0.5y ,<=600 Eq. 1
Labour hours 1.5x + 2y <=1,890 Eq. 2
2x <= y Eq. 3
x>=100 Eq. 4

Machine Labour
Available hours 715 2,250
Less: Hours required for confirmed order
Tables (40) (60)
Chairs (75) (300)
(115) (360)
600 1,890
Contribution margin per unit Table Chair
Sale price 2,300 900
Cost of sales
Material 1,000 300
Machine cost 450 225
Labour 90 120
Other manufacturing cost 200 50
1,740 695


560 205

















Objective function is to maximize 560x + 205y
x + 0.5y =600-------------------- Eq. 1 if y = 0, x = 600; if x = 0, y = 1,200
1.5x + 2y = 1,890 ----------------Eq. 2 if y = 0, x = 1,260; if x = 0, y = 945



Optimal solution is at points P,Q and R

Point P
Putting the value of x = 100 i.e. Equation 2, in Equation 4 we get:
can be ascertained by solving equation 2 and 4
150 + 2y = 1,890
y = (1,890 - 150) /2 = 870

Point Q
Multiplying Eq. 1 by 4, we get 4x +2y = 2,400 ------- Eq. 5
can be ascertained by solving equation 1 and 2
Subtracting Equation 5 from Equation 2, we get:
2.5x = 510
x = 204
Putting the value of x in Equation 1 we get:
204 + 0.5y = 600
y = 792

Point R
x + 0.5y = 600
can be ascertained by solving equation 1 and 3
2x = y OR x = 0.5y
0.5y +0.5y = 600
y = 600
Putting the value of y in equation 1 we get:
x + 300 = 600
x = 300

Putting the above values in objective function we get:
At point P: x = 100 560 = 56,000; y = 870 205 = 178,350; Total = 234,350
At point Q: x = 204 560 = 114,240 y = 792 205 = 162,360; Total = 276,600
At point R: x = 300 560 = 168,000; y = 600 205 = 123,000; Total = 291,000
Contribution is maximized at point R, profit = Rs. 291,000

(THE END)


MANAGEMENT ACCOUNTING
Suggested Answers
Final Examinations Winter 2010

Page 1 of 6


A.1 (a) (i) Budgeted cost and sales price per set Rupees
C & F value 9,500
Import related costs and duties 900
Variable cost of local value addition 3,500
Variable cost per set 13,900
Fixed production overheads (Rs. 12,000,000/5,000 sets) 2,400
Budgeted cost of production per set 16,300
Add: Gross profit (Rs. 16,300 25%) 4,075
Budgeted sales price per set to distributor 20,375

Budgeted gross profit (Rs 4,075 5,000 sets) 20,375,000
Less: Admin & selling expenses
Variable (Rs. 900 5,000 sets) (4,500,000)
Fixed (9,000,000)
Budgeted annual profit 6,875,000

(ii) Computation of budgeted consumer price of each set
Budgeted sales price of the company 20,375.00
Add: distributor margin (Rs. 20,375 10/90) 2,263.88
Budgeted sales price of the distributor 22,638.88
Add: wholesaler margin (Rs. 22,638.88 4/96) 943.29
Budgeted sales price of wholesaler 23,582.17
Add: retailers markup (Rs. 23,582.17 10%) 2,358.21
Budgeted retail price 25,940.39

Revised retail price (Rs. 25,940.39 95%) 24,643.37

Revised profit forecast after considering consultants recommendation:
Rupees
Sales (6,500 sets Rs. 24,643.37) 160,181,905
Less: Cost of goods sold for 6,500 units
Electronic Kits @ Rs 9,500 61,750,000
Cost of import and duty @ Rs 900 5,850,000
Local value addition @ Rs 3,500 22,750,000
Fixed overhead cost 12,000,000
(102,350,000)
Gross Profit 57,831,905

Less: Selling & Admin expenses
Variable (6,500 sets Rs 900) 5,850,000
Fixed 9,000,000
Cost of advertisement campaign 5,000,000
Cost of after-sale service (6,500 Rs. 450) 2,925,000
Retailers commission (Rs. 160,181,905 15%) 24,027,285
(46,802,285)
Profit by implementing the proposal of consultant 11,029,620


Based on above results, management should accept the recommendation of the
consultant.





MANAGEMENT ACCOUNTING
Suggested Answers
Final Examinations Winter 2010

Page 2 of 6


(b) In the light of the changes recommended by the consultant, the company will have to
consider whether it has the necessary infrastructure to:

(i) deal with a far larger number of retailers as against the present few distributors.
(ii) produce and sell extra 30% t.v. sets.


(iii) attend to after sale activities on its own. The question is silent as to who presently
attends to this activity.
(iv) conduct effective advertisement campaign.



Fixed expenses related to manufacturing as well as selling and admin are likely to
increase but no such increase has been anticipated.



A.2 Formula of learning curve
b
ax y=

Where
a is the labour cost for the 1
st

lot
x is the cumulative number of lots
b equals log(0.9)/log(2) = 0.152

Average cost Total cost

For 1,000 units (10 lots)
-0.152
10 20,000 y = 14,093.86 140,939

For 46,000 units (460 lots)
-0.152
460 20,000 y = 7,875.73 3,622,836

For 50,000 units (500 lots)
-0.152
00 5 20,000 y = 7,776.54 3,888,270

For 49,900 units (499 lots)
-0.152
499 20,000 y = 7,778.91 3,881,676
Cost for the last 100 units (500
th

lot)
6,594



45,000 units
(450 lots)
70,000 units
(700 lots)
Revenue 29,250,000 38,500,000

Material cost 13,500,000 21,000,000
Labour upto 460 lots 3,622,836 -
Labour upto 5oo lots - 3,888,270
Labour for additional 210 lots - 1,384,740
Less: labour upto 10 lots (140,939) (140,939)
3,481,897 5,132,071
Variable overheads 25% of labour cost 870,474 1,283,018
Fixed cost 1,300,000 1,300,000
19,152,371 28,715,089
Net profit 10,097 ,629 9,784,911

Conclusion: It will be more profitable to sell 45,000 units at a unit price of Rs. 650.









MANAGEMENT ACCOUNTING
Suggested Answers
Final Examinations Winter 2010

Page 3 of 6




A.3 (a) Carrying costs: Rupees
Interest @ 8.0% on Rs. 2,400 192
Insurance @1% on Rs. 2,400 24
216

Costs associated with each order:
Purchase department costs 3,000
Cost of delivery 6,000
9,000
Carrying costs per unit for special order:
Interest @ 8.0% on Rs. 2,400 x 95% 182.40
Insurance @1% on Rs. 2,400 x 95% 22.80
205.20

Determining stock-out and carrying costs:

Stock
level
Incremental
stock
Stock
out
Probability
Loss of
sales
Loss of
margin
(1,000x)
Carrying
costs
(216x)
Total
900 100 0.45 45 45,000
200 0.20 40 40,000
300 0.05 15 15,000
100,000 - 100,000
1,000 100 100 0.20 20 20,000
200 0.05 10 10,000
- 30,000 21,600 51,600
1,100 200 100 0.05 5 5,000 43,200 48,200
1,200 300 - - 64,800 64,800

Therefore, the company should reorder when the stock reaches 1100 units.
(b) Units Probability
Expected monthly sales 900 0.30 270
1,000 0.45 450
1,100 0.20 220
1,200 0.05 60
1,000
Expected yearly sales 12,000


( ) units 000 , 1 216 / 000 , 9 000 , 12 2 EOQ = =



Annual costs:
EOQ Special offer
Total cost Rupees

Cost of purchase
(2,400 x
12,000) 28,800,000 27,360,000 (2,400 x 95% x 12,000)
Ordering costs (9,000 x 12) 108,000 18,000 (9,000 x 2)
Carrying costs 1,000 / 2 x 216 108,000 615,600 6,000 / 2 x 205.20
29,016,000 27,993,600
Conclusion: FL should accept the offer





MANAGEMENT ACCOUNTING
Suggested Answers
Final Examinations Winter 2010

Page 4 of 6


A.4 (a) Return on capital employed Division A Division B
Divisional profit as per question 480,000 115,000

Capital employed:
Assets 2,433,100 643,000
Less: Liabilities 638,000 234,600
Net assets 1,795,100 408,400
Return on capital employed 26.74% 28.16%

Residual Income method
Net profit 202,400 48,900
Add: Apportioned head office cost 243,000 45,600
445,400 94,500
Return on equity (A 20%) 304,020 48,680
Residual Income 141,380 45,820

Equity(Assets-Liabilities) [2,433,100-913,000] [643,000-399,600] (A) 1,520,100 243,400

Division B has slightly higher return but on account of the far larger size of division A, this
small difference does not definitely indicate better performance from division B. Division
A is contributing 80% of the companys profit and seems more important for the company.

However, in the absence of information such as nature of business, nature and magnitude
of business risks and reasons for significant differences between debt equity ratios of the
two divisions, a meaningful comparison is difficult. It would be advisable for the company
to compare the returns with those of similar industries in the region.


(b) (i) Return on capital employed Rs. in 000
Net profit from new investment 6,000
Add: financial charges (90m 50% 14%) 6,300
Return on net investment 12,300

Net investment (80+10 16 being depreciation) 74,000
Return on capital employed 16.62%

Residual income Rs. in 000
Net profit on investments 6,000
Return on equity [29,000(W-1) 20%] 5,800
Residual income 200

W1:
Increase in assets 74,000
Less: increase in liabilities (90 2) 45,000
Increase in net assets 29,000

ROCE 25%




In view of low return i.e. 16.62% on new investment compared to existing return of
26.74%, the project will not be acceptable to the divisional manager.


However, the return of 16.62% pertains to the first year only. In future years, the
net investment would reduce and hence return percentage would increase
significantly. If the divisional manager takes a long term perspective, he may decide
to accept the proposal.
MANAGEMENT ACCOUNTING
Suggested Answers
Final Examinations Winter 2010

Page 5 of 6


In view of positive residual income, the project will be beneficial for the company.


Although ROCE and residual income are accepted as proper short-term measures
for large investment/disinvestment projects and for inter-division comparisons,
the evaluation must also consider NPV and DCF methods which take account of
time value of money as well as cover the entire life of the project.

(ii) Return on capital employed:
Impact on return
Contribution loss (9,000)
Reduced depreciation 20,000
Operating profit 11,000
Loss on sale of asset (12,000)
Net loss (1,000)

Capital employed:
Asset no change* -
Decrease in current liabilities (50% of 8.0 million) 4,000
Increase in capital employed 4,000





Residual income method:
Net loss before financial year as computed above (1,000)
Less: financial charges saved (14% of 4.0 million) 560
Net loss (440)
Increase in required return from equity (8,000**x20%) (1,600)
Residual income (2,040)



If the company follows a policy whereby gain/loss on sale of asset do not form part of
the divisions performance, the divisional profit would increase by Rs. 11 million
whereas the capital employed would increase by Rs. 4 million only and hence sale of
asset would be favourable from the divisional managers point of view.



However, if gain or loss on sale of asset is treated a part of divisional performance
then as a result of sale, the divisional profit would be reduced by Rs. 1.0 million
whereas capital employed would increase by Rs. 4.0 million. Hence in this case the
sale of asset would not be favoured by the divisional manager.



From the CEOs point of view, the sale would not be acceptable in any case as it would
contribute to a net reduction in residual income of the company by Rs. 2.04 million.


A.5 Since chemicals X & Y are produced from extracts P and Q which are produced in a joint
process and in a specified ratio, the production of each chemical is interdependent. Based on
the market demand, the company has 2 options as follows:

Production Option1: Produce 600,000 litres of product X and 160,000 Litres of product Y
(Working 1)

Production Option2: Produce 180,000 litres of product Y and 600,000 litres of product X and
dispose off any excess quantity of extract P in raw form D. (Working 2)
Working 1 Litres
Quantity of P required to produce 600,000 litres of chemical X (600,000 1/5) 120,000
Qty of Q produced in the joint process 1 (120,000 2/3) 80,000
Hence quantity of Y to be produced = (80,000 2) 160,000

MANAGEMENT ACCOUNTING
Suggested Answers
Final Examinations Winter 2010

Page 6 of 6


Working 2
Quantity of Q required to produce 180,000 litres of chemical Y (180,000 1/2) 90,000
Quantity of P produced in the joint process (90,000 3/2) 135,000
Quantity of P required to produce 600,000 litres of X being maximum demand for P 120,000
Surplus quantity of P to be sold without further processing 15,000

Option 1 Option 2
Litres Cost Litres Cost
Raw Material Costs
Quantity to be produced -P 120,000 135,000
Q 80,000 90,000
Total 200,000 225,000
Wastage (Total Qty / 0.2 0.8) 50,000 56,250
Inputs required in Process 1 (A) 250,000 281,250

Material A 2/3 166,667 4,166,667 187,500 4,687,500
Material B 1/3 83,333 3,333,333 93,750 3,750,000
Material C (Qty of Ext P 4) 480,000 36,000,000 480,000 36,000,000
Material D (Qty of Ext Q 1) 80,000 4,400,000 90,000 4,950,000
Total
Direct Labour - Process 1 80(A) 20,000,000 22,500,000
Direct Labour - Process 2 (50(600+160)) 38,000,000 (50(600+180)) 39,000,000
Variable overheads - Process 1 (57250) 14,250,000 (57281.25) 16,031,250
Variable overheads - Process 2 (32(600+160)) 24,320,000 (32(600+180)) 24,960,000
Total variable Costs 144,470,000 151,878,750

Option 1 Option 2
Profit & Loss
Sale of X (600000250) 150,000,000 150,000,000
Sale of Y (160000450)/(180000450) 72,000,000 81,000,000
Sale of P(15000100) 1,500,000
Sales revenue 222,000,000 232,500,000

Variable cost of production

(144,470,000)
(151,878,750
)
Fixed costs (5,000,000) (5,000,000)
Net profit 72,530,000 75,621,250

Option 2 produces higher profit.

(THE END)

MANAGEMENT ACCOUNTING
Suggested Answers
Final Examinations Summer 2010

Omair Jamal Page 1 of 7

A.1 Price Units Amount (Rs. 000s)
Men Women Men Women Men Women
Minimum 1,000 800 720,000 300,000 720,000 240,000
Maximum 4,000 2,500 120,000 50,000 480,000 125,000
Average 2,000 1,200 360,000 150,000 720,000 180,000
Total 1,200,000 500,000 1,920,000 545,000

Rs. 000s
Sales revenue gross (1,920,0000 +545,000) 2,465,000
Less : Commission to distributors 20% 30% of above 147,900
Cut size discount 40% (5% of 70%) 34,510
182,410
Sales net 2,282,590
Variable cost 100/220 of gross revenue 1,120,455
1,162,135
Less : Factory overheads 12 45m 540,000
Gross profit 622,135
Less : Admin overheads 12 15m 180,000
Cost of retail outlets 12 22 1.2m 316,800
496,800
Net profit 125,335


A.2 Objective function: Maximize Z =150,000x +100,000y

Current constraints:


120x +80y = 18200 Eq 1 if y =0, x 151; if x =0,y 227
80x +50y= 12000 Eq 2 if y =0, x =150; if x =0,y =240
x>0 and y>0
600x +400y= 91000 Eq 3 Eq 15
640x +400y 96000 Eq 4 Eq 28
40x= 5000
x= 125

80x +50y= 12000 Eq 2
10000 +50y= 12000
50y= 2000
y= 40

Revised constraints
120x +80y = 18400 if y =0, x 153; if x =0,y =230
80x +50y= 12000 Eq 2 if y =0, x =150; if x =0,y =240
x>0 and y>0
600x +400y= 92000 Eq 3 Eq 15
640x +400y 96000 Eq4 Eq2 8
40x= 4000
x= 100

80x +50y= 12000 Eq 2
8000+50y= 12000
50y= 4000
y= 80



MANAGEMENT ACCOUNTING
Suggested Answers
Final Examinations Summer 2010

Omair Jamal Page 2 of 7



Current Options:
Production
Contributions

x y
A 150 0 22,500,000
B 125 40 22,750,000
C 0 227 22,700,000

Required options:
Production
Contributions

x y
A 150 0 22,500,000
B 100 80 23,000,000
C 0 230 23,000,000

Shadow price for additional capacity: (23,000,000 - 22,750,000) = 250,000/200
=Rs.1,250 per hour

A.3 Cash Management
Total sales Units Weight Sales Ratio
Cash sales 25% 6,000 1.0 6,000
Credit sales 75% 18,000 1.1 19,800
24,000 25,800

Sales Revenue (Rs. in 000) 51,600
Cash Selling price per unit 2,000
Credit selling price per unit 2,200

Cash Requirement 2010 -11

Particulars
Qtr. 1 Qtr. 2
--- Rs. in 000 ---
Purchase of machinery (60,000) -
Sale receipts - -
Cash sales (2,000 6,000 / 4 94%) 2,820 2,820
Receipts from credit sales as per working below 5,211 9,120
Cost of goods sold variable (37,500 x 80%) /122 and 3 (5,000) (7,500)
Variable cost of finished stock 30,000 / 24,000 1,000 (1,250) -
Variable operating expenses (105 3 2,000) (630) (630)
Payment of fixed costs (457 2.5) / (457 3.0) (1,143) (1 ,372)

(59,992) 2,438



Month
1
st
Month
Qtr. 2
nd
Qtr.
1 2 3 4 5 6
---------- Rs. in 000 ----------
Working for credit sales

Credit sales (18,000122,200) 3,300 3,300 3,300 3,300 3,300 3,300
Settlement 70% 2,310 2,310 2,310 2,310 2,310
28% 924 924 924 924
Gross receipts 2,310 3,234 5,544 3,234 3,234 3,234 9,702
Tax @ 6% (333) (582)
Receipts net of tax 5,211 9,120


MANAGEMENT ACCOUNTING
Suggested Answers
Final Examinations Summer 2010

Omair Jamal Page 3 of 7

Operating expenses
Total operating expenses given 4,800
Less: Variable cost per unit (105 24,000) (2,520)
Bad debt expense (2,200 18,000 2%) (792)
Fixed operating expenses 1,488
Fixed cost
Fixed factory overheads 7,500
Less: Depreciation (60m 7.5m) / 15 (3,500)
Fixed operating overheads 1,488
5,488
Fixed cost per month 457


A.4 Noureen Industries Limited

Increased
commission 20%
Contribution Margin Own sales
department
------------Rs. in 000s-----------
Sales 80,000 80,000
Less: Variable expenses
Manufacturing costs 44,800 44,800
Sales commission 16,000 4,000
Finance cost 150 150
60,950 48,950
Contribution margin 19,050 31,050
Contribution margin as % of sales 23.8 38.8
Fixed expenses
Fixed overheads 6,500 6,500
Depreciation 700 700
Fixed admin costs 2,200 2,200
Finance cost 600 600
Fixed marketing costs 7,000
10,000 17,000

Equal net income level:
Let the required sales level be x.
Net operating income with increased commission =0.238x 10,000
Net operating income with own sales force =0.388x 17,000
Both will be equal at:
0.388x 17,000 =0.238x 10,000
0.15x =7,000
x =46,667
It would be beneficial for NIL to establish a full-fledged sales department if sales exceed Rs. 46,667,000.


A.5 Manufacturing cost of product B

Material A (105,000 250) 26,250,000
Material B (120,000 60) 7,200,000
(90,000 70) 6,300,000 13,500,000
Labour (W-1) [60 200 6 448 (W-1)] 32,256,000
Variable factory overhead [2006448(W-1) hours8] 4,300,800
Fixed factory overhead (W2) 4,425,000
Manufacturing cost of product B 80,731,800

MANAGEMENT ACCOUNTING
Suggested Answers
Final Examinations Summer 2010

Omair Jamal Page 4 of 7


W-1: Labour
Total working hours per labour (200 6) 1,200

Units produced in first 300 hours (300/6) 50
Units produced in remaining 900 hours (900/5) 180
No. of units produced per worker 230

No. of full units 100,000
Normal loss (5,000 60%) 3,000
103,000

No. of workers required (103,000 / 230) 448

W-2: Fixed overheads
Savings at 70% capacity for 6 months [6/12 (45.0 33.75)m] 5,625,000
Contribution from C - 10% capacity (70 60) for 6 months 1,200,000
A 6,825,000

Contribution from C - 40% capacity for 6 months B 4,800,000

Higher of A or B above 6,825,000
Less: contribution from C @ 20% capacity for 6 months 2,400,000
Opportunity cost of utilizing 20% capacity 4,425,000


A.6 (a) Most profitable option Costs
Product AMY at
Faisalabad
Product BNZ at
Lahore
Total Cost of 5,000 units 2,760,000 1,585,000
Total Cost of 1,500 units A 870,000 535,000
Variable cost of 3,500 units 1,890,000 1,050,000
Variable cost per unit 540 300
Variable cost for 1500 units B 810,000 450,000
Fixed cost A B 60,000 85,000

Option-1 If Lahore Factory purchases XPY from market for production of BNZ, contribution margin
from sale of BNZ could be as follows:

BNZ output
(units)
Revenue
Variable cost
excluding XPY
Cost of XPY Contribution margin
(1) (2) (3) (1)-(2)-(3)
1,500 1,800,000 450,000 1,087,500 262,500
3,000 3,480,000 900,000 2,175,000 405,000
3,500 3,920,000 1,050,000 2,537,500 332,500
4,000 4,320,000 1,200,000 2,900,000 220,000
5,000 5,150,000 1,500,000 3,625,000 25,000









MANAGEMENT ACCOUNTING
Suggested Answers
Final Examinations Summer 2010

Omair Jamal Page 5 of 7

In addition, contribution margin earned from AMY produced by Faisalabad and sold outside
could be as follows:
Product AMY output
(units)
Revenue Variable cost Contribution margin
1,500 1,275,000 810,000 465,000
3,000 2,475,000 1,620,000 855,000
3,500 2,800,000 1,890,000 910,000
4,000 3,100,000 2,160,000 940,000
5,000 3,500,000 2,700,000 800,000
Maximum Contribution Margin
Contribution margin from production of 3,000 units of BNZ by Lahore 405,000
Contribution margin from sale of 4,000 units in Faisalabad 940,000
Total contribution margin 1,345,000

Option-2 If Lahore Factory uses AMY produced by Faisalabad factory contribution margin from sale of
BNZ could be as follows:

BNZ output
(units)
Revenue
Variable cost
excluding AMY
Cost of AMY
Contribution
margin
(1) (2) (3) (1)-(2)-(3)
1,500 1,800,000 450,000 810,000 540,000
3,000 3,480,000 900,000 1,620,000 960,000
3,500 3,920,000 1,050,000 1,890,000 980,000
4,000 4,320,000 1,200,000 2,160,000 960,000
5,000 5,150,000 1,500,000 2,700,000 950,000

CM from in house production of 3,500 units 980,000
CM on production of 1,500 units in Faisalabad 465,000
1,445,000

To optimize profit, Lahore factory should use AMY produced by Faisalabad.

(b) Minimum price acceptable to Faisalabad Factory
Maximum contribution that could be earned by selling outside 940,000
Contribution earned by selling 1,500 units 465,000
Contribution to be earned by selling 3,500 units to Lahore 475,000
Profit to be earned per unit - 475000/3500 135.71
Variable cost per unit of AMY 540.00
Minimum price Faisalabad should charge for AMY 675.71

Maximum price acceptable to Lahore 725


A.6 (a)
Most profitable option
Costs
Product AMY at
Faisalabad
Marks
Product BNZ at
Lahore
Total Cost of 5,000 units 2,760,000 1,585,000
Total Cost of 1,500 units A 870,000 535,000
Variable cost of 3,500 units 1,890,000 1,050,000
Variable cost per unit 540 300
Variable cost for 1500 units B 810,000 450,000
Fixed cost A B 60,000 85,000

MANAGEMENT ACCOUNTING
Suggested Answers
Final Examinations Summer 2010

Omair Jamal Page 6 of 7


Option-1: Contribution margin earned from AMY produced by Faisalabad and sold outside could
be as follows:

Product AMY output
(units)
Revenue Variable cost Contribution margin
1,500 1,275,000 810,000 465,000
3,000 2,475,000 1,620,000 855,000
3,500 2,800,000 1,890,000 910,000
4,000 3,100,000 2,160,000 940,000
5,000 3,500,000 2,700,000 800,000

If Lahore Factory purchases 1,000 units of AMY from Faisalabad and balance units of XPY form
market for production of BNZ contribution margin from sale of BNZ could be as follows:


BNZ output
(units)
Revenue
Variable
cost
excluding
XPY
Cost of
AMY
(1,000
units
only)
Cost of XPY
(balance
units)
Contribution margin
(1) (2) (3) (4) (1)-(2)-(3)-(4)
1,500 1,800,000 450,000 540,000 362,500 447,500
3,000 3,480,000 900,000 540,000 1,450,000 590,000
3,500 3,920,000 1,050,000 540,000 1,812,500 517,500
4,000 4,320,000 1,200,000 540,000 2,175,000 405,000
5,000 5,150,000 1,500,000 540,000 2,900,000 210,000
Maximum Contribution Margin
Contribution margin from sale of 4,000 units in Faisalabad
940,000
Contribution margin from production of 3,000 units of BNZ by Lahore
590,000
Total contribution margin
1,530,000


Option-2 If Lahore Factory uses AMY produced by Faisalabad factory contribution margin from sale of
BNZ could be as follows:

BNZ output
(units)
Revenue
Variable cost
excluding AMY
Cost of AMY
Contribution
margin
(1) (2) (3) (1)-(2)-(3)
1,500 1,800,000 450,000 810,000 540,000

3,000 3,480,000 900,000 1,620,000 960,000

3,500 3,920,000 1,050,000 1,890,000 980,000

4,000 4,320,000 1,200,000 2,160,000 960,000

5,000 5,150,000 1,500,000 2,700,000 950,000


CM from in house production of 3,500 units 980,000
CM on production of 1,500 units in Faisalabad 465,000
1,445,000


To optimize profit, Faisalabad factory should sell 4,000 units outside and 1,000 units to Lahore factory at Rs.
540 per unit and Lahore factory should purchase balance units of XPY from market.






MANAGEMENT ACCOUNTING
Suggested Answers
Final Examinations Summer 2010

Omair Jamal Page 7 of 7

(b) Minimum price acceptable to Faisalabad Factory
Maximum contribution that could be earned by selling outside 940,000
Contribution earned by selling 1,500 units 465,000
Contribution to be earned by selling 3,500 units to Lahore 475,000
Profit to be earned per unit - 475000/3500 135.71
Variable cost per unit of AMY 540.00
Minimum price Faisalabad should charge for AMY 675.71

Maximum price acceptable to Lahore 725



(THE END)




MANAGEMENT ACCOUNTING
Suggested Answers
Final Examinations Winter 2009


A.1 Budgeted production
Budget production is 70% of the designed capacity
(150 tons 70% 24 hours 30 days 12 months) 907,200 tons

(A) Raw material
Quantity of raw material required
(1.25 tons 907,200 tons of finished product) 1,134,000 tons

Quantity of raw material for each shipment
(1,134,000 tons 12 of finished product) 94,500 tons

Total cost of purchases including transportation and other variable purchase
cost for each ton of product
(Rs. 2,500 62.4%) Rs. 1,560/ton

Per ton FOB price of raw material (Rs. 1,560 100 130) 1.25 Rs. 960

Total amount to be paid to supplier for each shipment (Rs. 960 94,500 tons) Rs. 90.72 mln

Credit period (45-30 days) 15 days

Trade credit: Average amount of liability (Rs. 90.72 million 15/30) Rs. 45.36 mln
Cost of consumables, spares and processing per ton (2,500 37.6%) Rs. 940

(B) Inventory Rs. in 000
Raw Material (94,500 tons 2 Rs. 960 1.3) 58,968
Work in progress (1,000 Rs. 960 1.3) +(1,000 940 50%) 1,718
Finished products (907,200 15 30 12) Rs. 2,500 94,500
Spares & consumables 20,000
175,186

(C) Debtors Rupees
Corporate clients
(40% 945,000 tons 10 360 Rs. 3,300 1.02) 35,343,000
Individual clients
(30% 945,000 tons 2 360 Rs. 3,300) (5,197,500)
Export clients
(30% 945,000 tons 5 360 Rs. 3,300 1.05) 13,643,438
43,788,938

Budgeted Sales quantity: Tons
Production during the year 907,200
Opening inventory 1/12 of above 75,600
Closing inventory 1/24 of the above (37,800)
Budgeted sales 945,000

Budgeted Price
Variable cost 2,500
Fixed Cost (Rs. 10.584 million 12 907,200 tons) 140
Total cost 2,640
Gross profit at 20% of selling price 660
Sales Price 3,300
(D) Other credit
Fixed cost (Rs. 10.584 million 15 / 30 days) 5,292,000
Other variable cost:
940 (Rs. 2,500 Rs. 1,560) 907,200 15 /360 days 35,532,000
40,824,000
MANAGEMENT ACCOUNTING
Suggested Answers
Final Examinations Winter 2009


Working Capital requirement
Average value of debtors 43,788,938
Average value of inventory 175,186,000
Average value of trade credit (45,360,000)
Average value of other credit (40,824,000)
132,790,938


A.2

Units
Average
time
Cumulative
time



1 40.00 40



2 38.00 76



4 36.10 144



8 34.30 274



16 32.58 521



32 30.95 990



64 29.40 1,882





No. of
workers
Available
hours*
Average time
per unit
Production
per worker
Total
production
No. of
Hours

40 522 32.58 16 640 20,880

7 992 30.95 32 224 6,944

153 1,882 29.40 64 9,792 287,946

153 206 28.00 7 1,071 31,518



11,727 347,288




Available hours: Up to March 31 174 x 3 522

Up to J une 21 174 x 5.7 992

Up to December 31 174 x 12 [1,882+206] 2,088



Cost of production


Units Rate Total cost


Materials 11,727 10,000 117,270,000


Labour 347,288 110 38,201,680


Overheads 11,727 4,000 46,908,000


202,379,680



Production (units)

11,727


Average cost per unit

17,258


Selling price per unit

21,573
Labour cost includes 10% bonus





























MANAGEMENT ACCOUNTING
Suggested Answers
Final Examinations Winter 2009




A.3 Standard weight of one unit of finished goods 11.88 kgs

Total input of raw material required for one unit of finished product {11.88 (100% 12%)} 13.50kgs

Standard material input: A:6.75kgs, B:4.50kgs, C:2.25kgs




Material A B C

A Year-end inventory Given kgs 1,014,200 754,000 228,000

B Ratio of inventory to material consumed Given % 11 13 8

C Material consumed A/B kgs 9,220,000 5,800,000 2,850,000

D Purchases during the year A+C kgs 10,234,200 6,554,000 3,078,000

E Purchases during :


Oct-Mar D/2 kgs 5,117,100 3,277,000 1,539,000

Apr-Sept D/2 kgs 5,117,100 3,277,000 1,539,000

F Value of year-end inventory Given Rs. 6,744,430 3,883,100 1,390,800

G Actual price per kg


Oct-Mar F/A Rs. 6.65 5.15 6.10

H Apr-Sept G/1.1 Rs. 6.05 4.68 5.55

J Average price Oct-Sept (G+H) / 2 Rs. 6.35 4.92 5.83

K Purchases during year J *D Rs. 64,987,170 32,245,680 17,944,740

L Material consumed at actual price K-F Rs. 58,242,740 28,362,580 16,553,940

M Standard price Given kg 6.40 4.85 5.90

N Standard cost C x M Rs. 59,008,000 28,130,000 16,815,000

P Price variance favourable/(unfavourable) L N Rs. 765,260 (232,580) 261,060

Total price variance Favourable

793,740


Mix variance

:
Raw
materia
l
Actual
quantity used
(kgs)
Standard mix of actual
quantity used
Actual
Variances
(kgs)
Standard
price per
kg
Variances (rupees)
Ratio kgs
A 9,220,000 3/6 8,935,000 (285,000) 6.40 (1,824,000)
B 5,800,000 2/6 5,956,667 156,667 4.85 759,835
C 2,850,000 1/6 2,978,333 128,333 5.90 757,165
17,870,000 17,870,000 NIL (307,000)

Yield Variance

:
Raw
materia
l
Standard mix
of actual
quantity used
Standard usage for
actual output (kgs)
Variances
(kgs)
Standard
price per
kg
Variances
Rs.
A 8,935,000 8,910,000 (25,000) 6.40 (160,000)
B 5,956,667 5,940,000 (16,667) 4.85 (80,835)
C 2,978,333 2,970,000 (8,333) 5.90 (49,165)
17,870,000 *17,820,000 (50,000) (290,000)
{Output 1.32 million units x standard input per unit 13.50 kgs(6.75 + 4.50 + 2.25kgs)}

Material usage variance (597,000) : Mix +yield variances











MANAGEMENT ACCOUNTING
Suggested Answers
Final Examinations Winter 2009




A.4 Cost of components without inspection:
Y Z

Total good components required (A) 10,000 10,000

Defectives expected (7/93 and 11/89 of 10,000) (B) 753 1,236

Total components to be purchased A + B = (C) 10,753 11,236




COSTS:


Purchase price of components @ 90 x (C) and 87 x (C) 967,770 977,532




Production cost of defective units:


Material cost at start - 50% of 420 (B) 158,130 259,560

Balance processing costs {B*60% of (720-210)} 230,418 378,216

Sale proceeds of defectives (B) x 200 (150,600) (247,200)

Total cost of components (including defective components and
defective units produced) 1,205,718 1,368,108




Cost of components with inspection:
Y Z

Total good components required (D) 10,000 10,000

Defectives expected B 10 (E) 75 124

Total components required D +E (F) 10,075 10,124




COSTS:


Purchase price of components @ 90 x (F) and 87 x (F) 906,750 880,788




Production cost of defective units:


Material cost at start - 50% of 420 x (E) 15,750 26,040

Balance processing costs (E) * 60% of (720 -210) 22,950 37,944

Sale proceeds of defectives (E) x 200 (15,000) (24,800)

Inspection cost @ Rs. 20 per component {20 x (C)} 215,060 224,720

Total cost of components (including defective components and
defective units produced) 1,145,510 1,144,692

Conclusion: The best option is that company should buy component Z and should carry out the inspection.


A.5 Option 1: Manufacturing all units at own factory
350,000 units 50,000 units 400,000 units

Rate
Amount
Rate
Amount Amount
Rs. in 000 Rs. in 000 Rs. in 000
Material units 24,000 8,400,000 24,000 1,200,000
Labour 3,400 1,190,000 6,800 340,000
Overheads 1,680 588,000 1,680 84,000
10,178,000 1,624,000
Existing fixed cost (260,000 x 1,120) 291,200
Less: Cost of idle labour (260,000 x 3,400 x 0.15/0.85 x 90%) (140,400)
150,800
Additional fixed costs (10% of 150,800) 15,080
Discount on material 2.5% of 9.6(8.4+1.2) billion (240,000)
Cost of producing 350,000/50,000/400,000 units 10,328,800 1,399,080 11,728,880








MANAGEMENT ACCOUNTING
Suggested Answers
Final Examinations Winter 2009





Option 2 Produce 350,000 units locally and import 50,000 units from Italy


Rs. in 000
Production of 350,000 units 10,328,800
Purchase of 50,000 units from outside @ 29,000 1,450,000
Total cost for 400,000 units 11,778,800

Option 3 Impact all (400,000) units from Italy

Purchase of 4000,000 units from outside @ 29,000 11,600,000
Add: fixed cost 150,800
Total cost 11,750,800


Decision
The company should produce 400,000 units at its own manufacturing facility.

A.6 PRICING OF NEW PRODUCTS

Calculation of expected sale
Pack size Total 500 grams 1 kg 2 kg

Units (a) 200,000 120,000 90,000

Total production (Kgs.) (b) 400,000 100,000 120,000 180,000

Percentage of total production 100% 25% 30% 45%

Consumption of Material A (Kgs) (c) 200,000 50,000 60,000 90,000

Cost of Material A {300 (c)} (d) 60,000,000 15,000,000 18,000,000 27,000,000

Material B {118.125(W-1) (a)} (e) 47,250,000 11,812,500 14,175,000 21,262,500

Packaging cost {(a) x 30, 40; 55} (f) 15,750,000 6,000,000 4,800,000 4,950,000

Labour {7.5625 (W-3) (b)} 3,025,000 756,250 907,500 1,361,250

Fixed overheads {9.375 (W-4) x (b)} 3,750,000 937,500 1,125,000 1,687,500

Total cost 129,775,000 34,506,250 39,007,500 56,261,250

Sales (cost +25%) 162,218,750 43,132,813 48,759,375 70,326,563

Sale price / unit

216 406 781

W-1 Material B Qty Rate Amount


Opportunity cost of 100,000 kgs (W-2) 100,000

12,000,000


Current disposal price of remaining available material 150,000 110 16,500,000


Purchase price of additional requirement 150,000 125 18,750,000


400,000 118.125 47,250,000

W-2 Opportunity cost of 100,000 kgs


Sale Price 100,000 160 16,000,000


Less: additional cost

(4,000,000)


12,000,000


Sale price if sold without processing

11,000,000


Higher of the above

12,000,000

W-3 Labour


Skilled Labour [(400,000 / 100 5) 70 1,400,000


Unskilled Labour (400,000 / 100 10) 45 1,800,000


Less: Skilled Labour - Idle hours now saved (5,000 70 /2) (175,000)


3,025,000


Cost per Kg 7.5625

W-4 Current fixed expenses 25,000,000 (100-25)% = Rs. 18,750,000


Production including new product (2,000,00050%)kgs +400,000 kgs = 1,400,000 Kgs.


Capacity utilization after introduction of new product = 70%
MANAGEMENT ACCOUNTING
Suggested Answers
Final Examinations Winter 2009




Fixed expenses (25,000,00090%) = 22,500,000


Additional fixed expenses on a/c of new product Rs. 3,750,000


Cost per Kg (for allocation purpose) Rs. 9.375
(The End)
MANAGEMENT ACCOUNTING
Suggested Answer
Final Examinations Summer 2009

Page 1 of 6

Ans.1 ABC LIMITED
Actual Jan-May 2009

Rupees
Sales (105,000x350) 36,750,000
Variable costs:
Raw materials (105,000x90) (9,450,000)
Direct labor (300 0.4) x 105,000 (12,600,000)
Other variable costs (300-112.50-120) x 105,000 (7,087,500)

Contribution margin 7,612,500

Revised Plan Jun-Dec 2009

LGV HGV Total
Sale price per unit 270 385.00
Variable cost:
Raw material cost
A (25x2x5/8) (31.25)
B (45x3x3/8) (45 3 2)/8 (33.75)
(65.00) (90.00)
Direct labor cost (3000.4) (120.00)
(120 0.6 1.1) (79.20)
Factory overhead cost (300-112.5-120) (67.50)
(67.5 0.9) (60.75)
Total variable cost (204.95) (277.50)
Contribution margin Rs 65.05 107.50

Sales mix ratio 2 1 3
Aggregate contribution margin Rs. 130.10 107.50 237.60

Fixed cost Jan-Dec:
Fixed cost for the year 25,000,000
Additional marketing cost 3,000,000
10% depreciation on machine cost J un-Dec 2009 70,000
28,070,000
Contribution recovered Jan-May 2009 (7,612,500)
Required contribution for J un-Dec 2009 20,457,500

Break even Sale quantity Jun-Dec 2009:
Break even quantity for:
High grade (20,457,500/237.60)

86,101

Low grade (86,101 2) 172,202
Break even Sale amount Jun-Dec 2009 Rs. 46,494,540 33,148,885 79,643,425










MANAGEMENT ACCOUNTING
Suggested Answer
Final Examinations Summer 2009

Page 2 of 6

Ans.2 Sales volume margin/profit/contribution variance
=7,125,000 / 50,000 1,500 =Rs. 213,750 (Fav) (W1)

Sales Price Variance
= 51,500 units (Rs. 540 Rs. 530) =Rs. 515,000 (Adv) (W2, 3, 4)

Raw material Price variance
=(Rs. 53 -50) 159,650 =Rs. 478,950 (Adv) (W6, 7, 8)

Raw material quantity variance
={159,650 (51,500 3)} 50 =Rs. 257,500 (Adv) (W6, 8, 9)

Labour rate Variance
=(Rs 75 Rs. 70) 51,500 2.625 hours =Rs. 675,937.50 (Fav) (W9, 10, 11, 12, 13)

Labour efficiency variance
=1/8 hour Rs. 75 51,500 =Rs. 482,812.50 (Adv) (W2, 12)

Variable overhead efficiency variance
=1/8 hour (24x51,500) =154,500 (Adv) (W14)

Variable overhead spending / expenditure variance
(24 22) 51,500 2.625 =Rs. 270,375 (Fav) (W14, 15)

W-1: Budgeted Sales quantity:
1,500 / 0.03 =50,000 units

W-2: Actual Sales quantity
50,000 +1,500 =51,500 units

W-3: Budgeted sale price:
27,000,000 / 50,000 =Rs. 540 per unit

W-4: Actual sale price:
540 10 =Rs. 530 per unit

W-5: Budgeted raw material quantity
=50,000 units 3 kgs =150,000 kgs

W-6: Budgeted material price
=7,500,000 150,000 kgs =Rs. 50 per kg (W5)

W-7: Actual material price
=Rs. 50 1.06 =Rs. 53 per kg

W-8: Total actual quantity used
=Rs. 8,461,450 Rs. 53 =159,650 kgs

W-9: Budgeted labour cost per finished unit
=9,375,000 50,000 =Rs. 187.50

W-10: Budgeted labour time for one finished unit

=[(Rs. 187.5) (Rs 50 150%)] =2.5 hours (W10)

W-11: Actual labour time taken for one finished unit
=2.5 +(18) =2.625 hours

MANAGEMENT ACCOUNTING
Suggested Answer
Final Examinations Summer 2009

Page 3 of 6


W-12: Budgeted labour cost per hour
=(Rs. 187.5 2.50 hours) =Rs. 75 per hour

W-13: Actual labour cost per hour
=(Rs. 9,463,125 (2.625 hours 51,500) =Rs. 70 per hour

W-14: Budgeted variable overhead rate per hour
3,000,000 / (50,000 2.50) =Rs. 24 per labour hour

W-15: Actual variable overhead rate per hour
2,974,125 / (2.625 51,500) =Rs. 22 per labour hour

RECONCILIATION OF BUDGETED CONTRIBUTION AND ACTUAL CONTRIBUTION

Rupees
Budgeted profit 7,125,000
Sales volume margin variance 213,750
Sale price variance (515,000)
Material price variance (478,950)
Material quantity (usage) variance (257,500)
Labour rate variance 675,937.50
Labour efficiency variance (482,812.50)
Variable overhead efficiency variance (154,500)
Variable overhead spending / expenditure variance 270,375
Actual profit 6,396,300


Patient days
of
occupancy
Diff from
Average (x)
Cost of
Supplies
Rs. 000
Diff from
Average (y)
Col 2 sqrd
x
2

(2) x (4)
Rs. 000
xy
Ans.3
1 2 3 4 5 6
Dec *8,370 -120 1,665 -55.0 14,400 6,600
J an 8,649 159 1,804 84.0 25,281 13,356
Feb 8,232 -258 1,717 -3.0 66,564 774
Mar 8,742 252 1,735 15.0 63,504 3,780
Apr 7,740 -750 1,597 -123.0 562,500 92,250
May 9,207 717 1,802 82.0 514,089 58,794
Total 50,940 0 10,320 1,246,338 175,554
Average 8,490 1,720

*8370 = 300 90% 31 days

Variable expenses = xy / x
2

Col 6 / Col 5 =175,554 / 1,246,338 =0.14086
=0.14086 1000 =Rs. 140.86 Variable rate per patient per day









MANAGEMENT ACCOUNTING
Suggested Answer
Final Examinations Summer 2009

Page 4 of 6

8

Alternate answer of Question No.3


Patient days
of occupancy
Cost of
supplies
1 x 2 Col 1 squared
1 (x) 2 (y) 3 (x*y) 4 (x
2
)
Dec 8,370 1,665 13,936,050 70,056,900
J an 8,649 1,804 15,602,796 74,805,201
Feb 8,232 1,717 14,134,344 67,765,824
Mar 8,742 1,735 15,167,370 76,422,564
Apr 7,740 1,597 12,360,780 59,907,600
May 9,207 1,802 16,591,014 84,768,849
Total 50,940 10,320 87,792,354 433,726,938

Variable Cost b

1 0.14085585
7,478,028
1,053,324
50,940) * (50,940 - 8 433,726,93 * 6
10,320) * (50,940 - 87,792,354 * 6
x) ( x n
y) x)( ( xy n
2 2
= = =

=




Variable cost =1,000 1 0.14085585 =Rs. 140.86 per day per patient

Ans.4 (a)



















Activity
Time in
days
EST EFT LST LFT
Total
Float*
0 1 2 0 2 0 2 0
1 2 8 2 10 8 16 6
1 3 10 2 12 2 12 0
2 4 6 10 16 16 22 6
2 5 3 10 13 22 25 12
3 4 3 12 15 19 22 7
3 6 7 12 19 12 19 0
4 7 5 16 21 22 27 6
5 7 2 13 15 25 27 12
6 7 8 19 27 19 27 0
* LFT-EST-Time in days
(b) The critical path is 0 1 3 6 7. The project duration is 27 days.


10
7
8
2
5 2

5
0 1
4
2
7
3
6
3
3
6
MANAGEMENT ACCOUNTING
Suggested Answer
Final Examinations Summer 2009

Page 5 of 6

Ans.5 Computation of budgeted gross profit based on:


Existing
budget
Rupees

Budget based on
recommended plan
Rupees

Material A (2 M kgs x 70% 32) / 0.92 48,695,652.16 (2 M kgs 70% x 32) / 0.984 45,528,455
Material B (2M x 30% 10) / 0.92 6,521,739.13 (2M 30% 10) / 0.984 6,097,561
Inspection cost (2M 0.50) / 0.984 1,016,260.00
Labour Cost (15 /60 2M Rs. 400/8) 25,000,000.00 (Rs.25m-Rs.3m (Note) 22,000,000.00
Variable overhead (2 M Rs. 5)/0.92 10,869,565.21 (Rs.5 x 80%) [(2M / (1-0.016)] 8,130,081.30
Fixed Overhead 4,000,000.00 (Rs. 4,000,00025%) 3,000,000.00
95,086,956.50 85,772,357.30
Savings 9,314,599.20

(Note) Savings in Labour Cost:

Average labour time for industry (15 minutes /1.25) 12 Minutes
Benefits of time saving
[(15 minutes 12 minutes) /60] 2 M 400/8 Rs. 5,000,000
Workers share (Rs. 5 million 40%) Rs. 2,000,000
Savings Rs. 3,000,000


Ans.6 Option I: Cost of short term loan per month:
Rate 18% per annum =1.5% per month

Cost of funds for 6 months ={10,000,000 (1.015)
6
}-10,000,000}= Rs. 934,433
Cost of funds for 1 month = 934,433 / 6 =155,739

Option II: Cost of financing through supplies:

Opportunity cost per month = 200,000 / 9,800,000 =2.04% / 2 =1.02% or Rs. 102,041 per month

Option III: Cost of factoring per month:
Rupees
Credit Sales 25,000,000 60/100 15,000,000
Interest charges 15,000,000 45/30 x 75% 1.25% 210,938
Fee 15,000,000 2% 300,000
Total Charges 510,938

Less : Savings in Bad debts and cost of credit control 200,000
Financial charges saving 63,641* (263,641)
247,297
Cost of funds =Rs. 303,547 per month

* Advance 75% of 15 million x 45/30 16,875,000
Less: interest charges (210,938)
Factors fees (300,000)
16,364,062
Less: requirement (10,000,000)
Overdraft reduction 6,364,062
Interest at 1% per month 63,641

Conclusion:

Option II is the cheapest option. The company should forego the cash discount of 2% and avail
credit for further 60 days.
MANAGEMENT ACCOUNTING
Suggested Answer
Final Examinations Summer 2009

Page 6 of 6


Ans.7 (a) X Y Z Total
Number of units A 10,000 2,000 50,000

Direct labour hours per unit B 2.5 5.0 2.8
Direct labour hours (AB) C 25,000 10,000 140,000 175,000
Total factory overheads D 1,400,000
Factory overhead rate per hour (D/C) E Rs. 8

Cost per unit - single factory overhead rate
method (B E) F 20 40 22.40

(b) Activity based costing
Set-up costs
Batch size G 125 50 10,000
Set-ups (A G) H 80 40 5 125
Set-up costs J 274,400 137,200 17,150 428,750

Production control
Machine hours per unit K 7.5 10.0 3.0
Total machine hours (A K) L 75,000 20,000 150,000 245,000
Production control M 75,000 20,000 150,000 245,000

Quality control Allocation
No. of inspections N 5% 5% 2%
Units inspected (A N) P 500 100 1,000
Hours per unit inspected Q 0.2 0.5 0.1
Total inspection hours (P Q) R 100 50 100 250
Quality control costs S 73,500 36,750 73,500 183,750

Materials management
No. of requisitions T 320 400 30 750
Material management costs U 156,800 196,000 14,700 367,500

Factory overheads General
Allocated on the basis of direct labour hours V 25,000 10,000 140,000 175,000

Total cost (J+M+S+U+V) W 604,700 399,950 395,350 1,400,000


Factory overhead cost per unit - activity
based costing (W A) Rs. 60.47 199.98 7.91

(The End)
MANAGEMENT ACCOUNTING
Suggested Answers
Final Examinations Winter 2008

Page 1 of 7
Ans.1 (a)
Material Labour
Variable
overheads
Total
Variable
Cost
Fixed
overheads
Total Cost
Equivalent units:

Completed units
(6,000 +1,000 800) 6,200 6,200 6,200 6,200

Closing work-in-progress 1,200 600 600 600
Opening work-in-progress (500) (200) (200) (200)
Total equivalent units 6,900 6,600 6,600 6,600

Total cost (Rs.) 83,490,000 14,256,000 10,890,000 108,636,000 17,490,000 126,126,000
Cost per unit (Rs.) 12,100 2,160 1,650 15,910 2,650 18,560

b

(i) Absorption costing profit statement:
Rupees
Sales (6,000 24,000) 144,000,000
Op WIP 6,700,000
Op finished goods (17,000 800) 13,600,000
Production cost 126,126,000
Closing WIP (18,396,000)
Closing finished goods stock (18,560 1,000) (18,560,000)
109,470,000
Gross profit 34,530,000
Less: variable selling and administration costs (1,600
6,000) 9,600,000
Fixed selling and administration costs 12,000,000
21,600,000
Net profit 12,930,000

(ii) Marginal costing profit statement:
Rupees
Sales 144,000,000
Opening WIP 6,200,000
Opening finished goods (800 x 14,500) 11,600,000
Variable cost of production 108,636,000
Closing WIP (16,806,000)
Closing finished goods stock (1,000 x 15,910) (15,910,000)
Variable cost of sales 93,720,000
Variable selling and administration costs (1,600 6,000) 9,600,000
103,320,000
Contribution 40,680,000
Less: Fixed costs (17,490 +12,000) 29,490,000
Net profit 11,190,000

Working
Closing work-in-progress (Rs.) 14,520,000 1,296,000 990,000 16,806,000 1,590,000 18,396,000
Cost per unit last year 11,000 2,000 1,500 14,500 2,500 17,000
Opening work-in-progress (Rs.) 5,500,000 400,000 300,000 6,200,000 500,000 6,700,000









MANAGEMENT ACCOUNTING
Suggested Answers
Final Examinations Winter 2008

Page 2 of 7
Ans.2 (a) ROI = net profit / total assets (investment)

Computation of net profit
Rs. in million
Annual profit before depreciation and financial charges 150
Depreciation [(Rs. 500 M - 20 M) / 10 years] (48)
Financial Charges (Rs. 600 70% 10%) (42)
60

Computation of net capital employed (mid year) for year 2009
Rs. in million
Net Book Value at 1
st
J anuary, 2009 500
Net Book Value at 31
st
December, 2009 [(480 9/10) +20] 452

Mid Year Value for year 2009 [(500 +452) /2] 476
Working Capital 100
Average net capital employed 576

ROI for the year 2009 [(Rs. 60M / Rs. 576M) 100] 10.42%

Computation of average net capital employed (mid year) for year 2015
Rs. in million
Net Book Value at 1
st
J anuary, 2015 [(480 4/10) +20] 212
Net Book Value at 31
st
December, 2015 [(480 3/10) +20] 164

Mid Year Value for year 2015 [(212 +164) /2] 188
Working Capital 100
Average net capital employed 288

ROI for the year 2015 [(Rs. 60 / Rs. 288) 100] 20.83%

(b) Comments on appropriateness of the result

1. ROI method focuses on short term performance whereas investment decision should be
evaluated on the life of the project.
2. Although the net profit for the years 2009 & 2015 are same but the ROI is much higher in
2015 as compared to 2009 which shows that it is not an appropriate ratio for comparing the
performance on year to year basis.

Ans.3 Total Production Capacity
Kgs
Model A (7,500 x 80) 600,000
Model Z (7,500 x 100) 750,000

Computation of per kg cost (Model A)
Per Unit Cost Total Cost
Rupees Rupees
Raw Material Cost (400 / 0.85) 470.59 282,354,000
Natural Gas (0.5 MMBTU Rs. 80) 40.00 24,000,000
Electricity (2 KWH 12 Rs.) 24.00 14,400,000
Water (5 gallons Rs. 2) 10.00 6,000,000
Plant depreciation (33,000,000 / 600,000) 55.00 33,000,000
Labour cost (30,000,000/600000) 50.00 30,000,000
Other production overhead (60,000,000 / 600) 100.00 60,000,000
749.59 449,754,000

MANAGEMENT ACCOUNTING
Suggested Answers
Final Examinations Winter 2008

Page 3 of 7


Computation of per kg cost (Model Z)
Per Unit Cost Total Cost
Rupees Rupees
Raw Material Cost 400.00 300,000,000
Natural Gas (0.4 MMBTU Rs. 80) 32.00 24,000,000
Electricity (1.5KWH Rs. 12) 18.00 13,500,000
Water (4 gallons Rs. 2) 8.00 6,000,000
Plant depreciation (37,500,000 / 750,000) 50.00 37,500,000
Labour Cost (33,000,000/600) 55.00 41,250,000
Other production overheads (80,500,000 / 750,000) 107.33 80,500,000
670.33 502,750,000
Wastage (10/90 x 670.33) 74.48 55,860,000
744.81 558,610,000

Other production overheads for Model Z
Rupees
Fixed cost (40% x 70.0 million) 28,000,000
Variable cost (70 million x 60% 75 / 60) 52,500,000
80,500,000

Selling and administration expenses for Model Z
Fixed cost (60% x 45 million) 27,000,000
Variable cost (45 million x 40% 75 / 60) 22,500,000
49,500,000

Computation of financial charges
Rs. in million Rs. in million
Investment Size
Plant Cost 660.000 750.000
Working capital 108.000 135.000
768.000 885.000

60% Debt 460.800 531.000

Annual Financial Charges @ 12% 55.296 63.720

Profitability Analysis of Model A and Model Z


Model A Model Z
Rupees Rupees
Sales @ Rs. 900/ Kg 540,000,000 675,000,000
Cost of goods sold (449,754,000) (558,610,000)
Gross Profit 90,246,000 116,390,000
Admin and selling overheads (35,000,000) (49,500,000)
Financial Charges (55,296,000) (63,720,000)
Net Profit (50,000) 3,170,000
Tax @ 30% - (951,000)
(50,000) 2,219,000

Equity (768 40%) 307,200,000 (885 40%) 354,000,000

Return on equity (0.02) %) 0.63%
Model Z is to be preferred over Model A.
MANAGEMENT ACCOUNTING
Suggested Answers
Final Examinations Winter 2008

Page 4 of 7


Ans.4 (a)

Existing
Assets/Liabilities
Proposed
level
Rupees Rupees
Debtors 360,000 x 160 57,600,000 360,000 x 160 x 1.25 x 2 144,000,000
Stocks 360,000 x 120 x 60% 25,920,000 360,000 x 120 x 60% x 1.25 32,400,000
Creditors 2/3 of above (17,280,000) 2/3 of above (21,600,000)
Finished goods 360,000*120*2 86,400,000 360,000 x 120 x 2 x 1.25 108,000,000
152,640,000 262,800,000

Rupees


Increase in working capital (Rs. 262,800,000 Rs. 152,640,000) 110,160,000




Cost of funds @ 16% of above 17,625,600




Profit margin on extra sales 360,000*0.25*40*12 43,200,000


Extra profits are more than 2.4 times the cost of funds; hence the proposed credit policy is feasible.

(b) Cost of factoring per month
Rupees
Fee (8,000,000 x 80% x 2% 128,000
Commission (8,000,000 x 30/20 x 4%) 480,000
608,000
Less : Savings in management costs (600,000 / 12) (50,000)
Savings on bad debts (8,000,000 x 30/20 x 1%) (120,000)
438,000

Cost of short term finance from bank, per month
Rupees
Interest (8,000,000 x 0.8 x 18% / 12 ) 96,000
Processing fee (3% x 8,000,000 x 80%) 192,000
288,000

Obtaining short term loan facility is less costly and hence a better option.


Ans.5 Computation of labour hours required

Assuming that the learning curve rate is x:
800 4 x x =2312
x
2
=2312 / 3,200
x

=0.85


Batches Cumulative quantity
Cumulative average
hours per unit
Cumulative hours
1 40 20 800
2 80 17 1,360
4 160 14.45 2,312
8 320 12.28 3,930
16 640 10.44 6,682

Hence, additional hours for 480 units =6,682 2,312 =4,370 hours

MANAGEMENT ACCOUNTING
Suggested Answers
Final Examinations Winter 2008

Page 5 of 7



Labour hour rate:
Rupees
600 normal hours +200 overtime hours 800,000
600 +200 x 2 800,000
1,000 hours 800,000
Hourly rate 800

Direct labour:

Hours
Direct
labour cost
Rupees
8 workers for 10 weeks for 40 hours 3,200 @ Rs. 800 per hour 2,560,000
2 workers for 4 weeks for 40 hours 800 @ Rs. 800 per hour 256,000
Overtime 370 @ Rs. 1,600 per hour 592,000
4,370 3,408,000

Incremental cost of producing 480 units:
Amount in Rs
Direct materials (480 10,000) 4,800,000
Direct labour 3,408,000
Variable overhead (4,370 500) 2,185,000
10,393,000

Cost per unit (10,393,000/480) 21,652

Hence, quotation can be accepted at Rs 25,000 per unit.


Ans.6 Computation of Sales for 2008

A
B
Normal
B
Corporate Total
Ratio of sale price 1.00 1.60 1.44
Actual sale Qty 5,400.00 2,880.00 720.00
Ratio of sale value 5,400.00 4,608.00 1,036.80 11,044.80
Sales value 2,700,000.00 2,304,000.00 518,400.00 5,522,400.00

A B
Current years production (at 90 % capacity) 5,400.00 3,600.00

Production at full capacity 6,000.00 4,000.00

If only B is produced the company can produce 9,000 units (4,000 +6,000 / 1.2).
Required production of B in the next year =(2,880 x 1.3) +(2 x 720) =3744 +1440 =5,184 units
Remaining capacity can be utilised to produce 4,579 units of A [(9,000 - 5,184) x 1.2].

Computation of Sales for 2009
Rupees
Sales of A (4,579 x 500) 2,289,500
Sales of B (5,184 x 800) 4,147,200
6,436,700
Discount to Corporate customer (1,440 800 15%) 172,800
6,263,900

MANAGEMENT ACCOUNTING
Suggested Answers
Final Examinations Winter 2008

Page 6 of 7



Consumption of Raw Material
Kgs
Consumption of raw material in 2008 (A: 5,400 x 2.4 / 0.96) 13,500.00
Consumption of raw material in 2008 (B: 3,600 x 2.4 / 0.90) 9,600.00
Total 23,100.00

Rupees
Price per kg of raw material ( 2,310,000 / 23,100) 100.00

Total expected consumption in 2009 (A: 4,579 x 2.4 / 0.96) 11,447.50
Total expected consumption in 2009 (B: 5,184 x 2.4 / 0.90) 13,824.00
Total consumption for 2009 25,271.50

Average price for 2009 ((100 x 3) +(110 x 9)) / 12 107.50

Total cost of raw material for 2009 2,716,686.25

Computation of Direct Labour
Hours
Labour hours used in 2008 (A: 5,400 5) 27,000
Labour hours used in 2008 (B: 3,600 6) 21,600
48,600

Labour hours forecast for 2009 (A: 4,579 5) 22,895
Labour hours forecast for 2009 (B: 5,184 6) 31,104
53,999

Increase in labour hours 5,399

Labour cost for 2009 (1.15 x (777,600 x 53,999 / 48,600) Rs. 993,582





Production overheads for 2008 :
Rupees
Fixed overheads (40% x 630,000) 252,000.00

Variable overheads (630,000-252,000) 378,000.00

A B Total
Ratio of variable overheads 1.00 2.00
Total units produced 5,400.00 3,600.00
Product (units) (K) 5,400.00 7,200.00 12,600.00

Total variable overheads (Rs.) (L) 162,000.00 216,000.00 378,000.00

Per unit variable overheads (Rs.) (L /K) 30.00 60.00



MANAGEMENT ACCOUNTING
Suggested Answers
Final Examinations Winter 2008

Page 7 of 7
Production overheads for 2009:
A B Total
Fixed overheads (1.05 x 252,000) (Rs.) 264,600.00
Per unit variable overheads (Rs.) 33.00 66.00
Total units 4,579 5,184
Total variable overheads (Rs.) 151,107.00 342,144.00 493,251.00
Total overheads (Rs.) 757,851.00

PROFIT FORECAST STATEMENT FOR 2009

Rupees
Sales 6,263,900.00
Material 2,716,686.25
Labour 993,582.00
Overheads 757,851.00 4,468,119.25
Gross margin 1,795,780.75

Selling and administration expenses (800,000 x 1.1) +250,000 1,130,000.00
665,780.75

(THE END)
MANAGEMENT ACCOUNTING
Suggested Answer
Final Examinations Summer 2008



Page 1 of 8
Ans.
1 (a) The company has to supply minimum sales to the customer as follows:


Unit Rate Rs.
X 40,000 900 36,000,000
Y 96,000 1,200 115,200,000
151,200,000


Further sales possible (200,000,000 151,200,00) 48,800,000


X Y
Contribution per unit Rs. 260 170
Contribution per hour Rs. 2,080 1,020
Contribution % on sales 29 14


X contributes more than Y.
Therefore, 48,800,000 / Rs.900 = 54,222 units of X should be produced.

Check whether this level of production can be attained in available hours:


Units Hours
X (40,000+ 54,222 ) 94,222 11,778
Y 96,000 16,000
27,778


Therefore, maximum contribution / profit will be as follows:
X Y Total - Rs.

Sales in unit 94,222 96,000
Contribution per unit 260 170
Total contribution 24,497,720 16,320,000 40,817,720

(b) Increase / (decrease) in profit if the loan is taken

Extra Sales of X if loan is taken (60 mln / 900) 66,667 units
Production possible in remaining hours (6,222* x 8) 49,776 units
Contribution on 49,776 units (49,776 x 260) Rs. 12,941,760
Bank charges on Rs.25 mln at 16% 4,000,000
Additional contribution if bank facility availed 8,941,760

*(34,000 27,778)


Ans.
2 Computation of working capital
Rupees
Debtors:
Exports (D*30/360) (Working 1) 40,916,667
Local customers with 2% discount (F*0.98*10/360) (Working 2) 6,533,333
Local customers with 1% discount (E*0.99*20/360) (Working 2) 6,600,000
Local customers who do not avail discount (C-100,000,000-E-F-13180000)/12 23,818,333

Advance against raw material C (N x 15/360x0.5) (Working 3) 2,043,215
MANAGEMENT ACCOUNTING
Suggested Answer
Final Examinations Summer 2008



Page 2 of 8


Closing Stock
Raw material (H) (Working 3) 48,342,000
Finished Goods (P) (Working 4) 77,508,667

Creditors - Raw material A (L x 30/360) (24,518,583)
Raw material B (M x 45/360) (24,518,583)
Labour, FOH and Admin Expenses (B x 0.38 x 15/360) (19,791,667)

136,933,382
Working 1
Sales in Previous year A 1,000,000,000
Sales in Current year ( A x 1.25 ) B 1,250,000,000
Local sales ( A x 60%x1.1x1.15 ) C 759,000,000
Exports (B-C) D 491,000,000

Working 2
Assume sale with 1% discount = X
Sale with 2% discount will be = 2X

Discount = 0.01X+(0.02*2X) = 0.05 X = Rs. 6,000,000

Therefore sale on which 1% discount will be given = X = 6,000,000/0.05 E 120,000,000
Therefore sale on which 2% discount will be given = 2X = 120,000,000*2 F 240,000,000

Working 3
Local sales at last year's price (1 billion * 60% * 1.1) 660,000,000
Exports as above 491,000,000
Total Sales excluding the effect of price increase G 1,151,000,000

Purchases of Raw Material

Closing stock of Raw material (G*0.48*1.05/12) H 48,342,000
Raw material included in cost of sales (G*0.48*1.05) I 580,104,000
Opening stock of Raw material (A*0.48/12) J (40,000,000)

Total raw material purchases K 588,446,000



Purchases of A (K*3/6) L 294,223,000
Purchases of B (K*2/6) M 196,148,667
Purchases of C (K*1/6) N 98,074,333

Working 4

Raw material as above (H) 48,342,000
Labour and factory overheads (B*28%/12) 29,166,667

P 77,508,667








MANAGEMENT ACCOUNTING
Suggested Answer
Final Examinations Summer 2008



Page 3 of 8

Ans.
3
(a) (70% = Rs. 2.8 bn)

(A) Fav = 60% = -0.5 bn

(30% = Rs. 0.8 bn)
(Research = -0.1 bn)
(20% = Rs. 0.7 bn)

(B) Unfav = 40% = -0.5 bn

(80% = Rs. -0.4 bn)

(40% = 2.0 bn)

(No research = -0.5 bn) (35% = 1.2 bn)
(C)
(25% = -0.2 bn)

Rs.
billion


0.7


0.6
0.3

0.5 bln
0.2
0.4


0.8

0.4


0.35

0.5 bln

0.25




= Decision point (DP)




= Chance point (CP)

(i) At CP2, EV = (0.7 2.8) + (0.3 0.8) = 1.96 + 0.24 = 2.2 billion
(ii) At CP3, EV = (0.2 0.7) + (0.8 0.4) = 0.14 0.32 = 0.18 billion
(iii) At CP1, EV = (0.6 2.20) + (0.4 0.18) = 1.32 0.072 = 0.1248 billion
(iv) At DP B, EV = 1.248 0.5 = 0.748 billion
(v) At DP A, EV = 0.748 01 = 0.648 billion
(vi) At CP4, EV = (0.4 2.0) + (0.35 1.20) 0.25 0.2) = 0.8 + 0.42 0.05 = 1.17
billion
(vii) At DP C and A, EV = 1.17 0.5 = 0.670 billion
The companys profits would be higher by Rs. 220 million (0.670 billion 0.648 billion)
if it did not carry out research.
No research





Undertake research
0.1 billion
Initial launch
Initial launch
Feasible
Not feasible
2.8
0.8
0.7
-0.4
2.0
1.2
-0.2
MANAGEMENT ACCOUNTING
Suggested Answer
Final Examinations Summer 2008



Page 4 of 8

(b) Expected profit if the research is carried out
Profit expectation at point A = (2.2 x 70% x 60%) + (0.2 x 30% x 60%)
= 0.924+ 0.036
= 0.960 billion

Profit expectation at point B = (0.1 x 0.2 x 0.4) (1.0 x 80% x 40%)
= (0.008 - 0.32)
= -0.312

Profit expectation if research is carried out = 0.960 0.312 = 0.648 bln

Expected profit if research is not carried out (Point C)
(1.5 x 40%) + (0.7 x 35%) (0.7 x 25%) = 0.6 + 0.245 0.175
= 0.67bn

The company should not carry out research, as then it could earn higher profit of Rs. 22
million (670 648).




Ans.
3
(b)
No. of units
Average
hours
Total hours
1 5,000 5,000
2 4,000 8,000
4 3,200 12,800
8 2,560 20,480
16 2,048 32,768


Hours used for 9-16 units (32768-20480) 12,288

Cost of labour 12288 x 100 Rs. 1,228,800

Direct materials 8*400000 Rs. 3,200,000

Variable overheads 12288 x 80% Rs. 983,040
5,411,840

Margin 1,352,960

Sale price Rs. 6,764,800


Ans.
4 Computation of Units Sold
Rupees

Actual Sales Price per unit (100 / 1.0526) 95
Sales price variance per unit (100 95) (A) 5
Adverse Selling Price Variance (B) 24,250,000
Units Sold during the period B / A 4,850,000

Computation of Units Manufactured
Million units
Units Sold 4.85
Increase in inventory level 0.23
Units Manufactured 5.08

MANAGEMENT ACCOUNTING
Suggested Answer
Final Examinations Summer 2008



Page 5 of 8





(a) Actual cost of raw materials consumed (million rupees)



Standard cost Price variance
Mix
variance*
Actual cost

X (5.08*15) 76.2000 2.2950 (7.6200) 70.8750
Y (5.08*20) 101.6000 (2.7030) 6.0960 104.9930
Z (5.08*27) 137.1600 (3.7995) (0.9140) 132.4465
314.9600 (4.2075) (2.4380) 308.3145

*See mix variance working

(b) Material mix variance


Standard mix
(millions Kgs)
Actual mix
(million Kgs)
Difference
(million Kgs)
Variance
(million Rs.)

X (5.08*5) 25.40 22.860 2.540 7.620
Y (5.08*10) 50.80 53.848 -3.048 -6.096
Z (5.08*15) 76.20 75.692 0.508 0.914
152.40 152.400 2.438

(c) Labour Cost Variance


Quantity
consumed
million Kgs
Actual
labour at
standard
cost
Standard
labour cost
Labour cost
variance
million Rs.


skilled 22.860 22.860 25.400 2.540 Fav
semi-skilled 53.848 40.386 38.100 -2.286 Adv
unskilled 75.692 7.569 7.620 0.051 Fav
152.400 71.094 71.400 0.306






Ans.
5
Activity
1 2 3 4
Unallocated Total

(a)
Manufactur-
ing
Customer
service
Order
process-
ing
Ware-
housing

Indirect labour 4,320,000 1,440,000 - 1,440,000 - 7,200,000

Other manufacturing
overheads 8,550,000 - - 450,000 - 9,000,000
Quality Control 900,000 600,000 - - - 1,500,000
Transportation 126,000 882,000 - 252,000 - 1,260,000
Admin salaries - - 600,000 750,000 1,650,000 3,000,000
13,896,000 2,922,000 600,000 2,892,000 1,650,000 21,960,000


Budgeted activity
level 72,000 120 20 40,000,000

Cost driver rate 193.00 24,350.00 30,000.00 0.0723



per labour
hour
per order
day
per order
processed
per Re. of
material
usage


MANAGEMENT ACCOUNTING
Suggested Answer
Final Examinations Summer 2008



Page 6 of 8




(b) Order by KSL
Costs under present method

Direct material cost 3,000,000
Direct labour 1,500,000
Factory overheads (90% of direct labour) 1,350,000
5,850,000
Mark-up - 50% 2,925,000
Sale price (A) 8,775,000

Costs under ABC Method

Direct material cost 3,000,000
Direct labour 1,500,000
Other manufacturing cost (6,000 x 193) 1,158,000
Customer service (10 x 24,350) 243,500
Order processing (1 x 30,000) 30,000
Warehousing (3,000,000 x 0.0723) 216,900
6,148,400
Margin -20% of sales price 1,537,100
Sale price (B) 7,685,500

Discount that may be allowed (A-B) 1,089,500


Ans.
6 (a) EARNINGS UNDER PROPOSED OPTION
Total billing to customers (working 1) 9,000,000

Less: Cost of raw material used after warranty period (working 3) 2,400,000
Cost of labour & variable overhead (3.8 mln + 10%) (working 2) 4,180,000
Salary of Supervisor 480,000
Increase in other fixed overheads 720,000
7,780,000
Net profit excluding cost of material used during warranty period 1,220,000

LESS: EARNINGS UNDER THE PRESENT OPTION
Mark-up earned on supply of material 360,000
Share of billing received from AHA 990,000
1,350,000

Less: Payment to AHA for services provided during
warranty period (760,000+30%) (working 2) 988,000
362,000
Net savings 858,000

Working 1


Domestic
Customers
Industrial
Customers
Total

Ratio of services provided by AHA A 20 80 100
Share of KL in % B 15.00% 10.00% N/A
Ratio of KL's share C (A*B) 3 8 11
MANAGEMENT ACCOUNTING
Suggested Answer
Final Examinations Summer 2008



Page 7 of 8
Annual share received from AHA D 270,000 720,000 990,000
Total billing by AHA E (D/B) 1,800,000 7,200,000 9,000,000


Working 2
Net recoveries from customers F (E-D) 1,530,000 6,480,000 8,010,000

Less: Recoveries in respect of material
(See working 3) G 3,450,000


Recoveries in respect of services
(labour & overheads) H (F-G) 4,560,000


Cost of labour and overhead incurred by AHA
(after warranty period) J (H*100/150) 3,040,000


Cost of labour and overhead incurred by AHA
(during warranty period) K (J*20/80) 760,000

Total cost of labour and overhead L(J + K) 3,800,000

Working 3
Mark-up charged by KL on material billed to AHA 360,000
Cost of material despatched (for use after warranty period) 2,400,000
2,760,000
Mark-up charged by AHA on material billed to customers (2,760,000*0.25) 690,000
Total billing in respect of material 3,450,000

Alternative Answer 6(a)

Savings/additional revenues if services provided by KL

Mark-up charged by AHA, from the customers on cost of
material (working 1) 690,000


Mark-up charged by AHA from KL on services provided
during warranty period
L (K*0.3)
(working 2) 228,000


Mark-up charged by AHA, from the customers on cost of
labour and overhead
M (H-J)
(working 2) 1,520,000

Total 2,438,000

Less: Additional costs and decline in revenues

Increase in cost of labour and variable overheads N ((J+K)*0.1)
(working 2)
380,000
Supervisors salary 480,000
Increase in other fixed overheads 720,000
1,580,000
Net savings 858,000

Working 1
Mark-up charged by KL on material billed to AHA 360,000
Cost of material ispatched (for use after warranty period) 2,400,000
2,760,000
Mark-up charged by AHA on material billed to customers 2760000*0.25 690,000
Total billing in respect of material 3,450,000


MANAGEMENT ACCOUNTING
Suggested Answer
Final Examinations Summer 2008



Page 8 of 8

Working 2

Domestic
Customers
Industrial
Customers
Total

Ratio of services provided by AHA A 20 80 100
Share of KL in % B 15.00% 10.00% N/A
Ratio of KLs share C (A*B) 3 8 11
Annual share received from AHA D 270,000 720,000 990,000
Total billing by AHA E (D/B) 1,800,000 7,200,000 9,000,000
Net recoveries from customers F (E-D) 1,530,000 6,480,000 8,010,000
Less: Recoveries in respect of material G (see working 1) 3,450,000

Recoveries in respect of services
(labour & overheads) H (F-G) 4,560,000


Cost of labour and overhead incurred by AHA
(after warranty period) J (H*100/150) 3,040,000


Cost of labour and overhead incurred by AHA
(during warranty period) K (J*20/80) 760,000

Total cost of materials L(J + K) 3,800,000


(b) (i) It might be beneficial for Kamran Limited (KL) to focus on core business rather
than on non-core areas like after-sale service.
(ii) Ahmed Hasan Associates (AHA) might be technically more competent at
providing these services.
(iii) KL should also consider the reliability of AHA as an outside supplier of these
services. If after-sale service is a critical component of KLs business, it might be
better to do it in-house.
(iv) There is a potential for KL to be inefficient in terms of cost control during parts
production since the company charges a cost-plus margin to AHA. There is not
much incentive for KL to control costs.
(v) The numbers provided by the cost accountant might be misleading since these are
predominantly direct costs of providing the service and possible effects on other
overheads may not have been considered.

(THE END)

Vous aimerez peut-être aussi